ORTHOPEDIC MCQS WITH ANSWER ANATOMY 05
ORTHOPEDIC MCQS WITH ANSWER ANATOMY 05
1. A 65-year-old man with ankylosing spondylitis sustains an extension injury to his cervical spine. Two days later, a progressive neurologic deficit develops at the C6 level. An MRI scan is shown in Figure 1. What is the most likely diagnosis?
1- Epidural hematoma
2- Herniated disk
3- Tumor
4- Extruded epidural bony fragment
5- Abscess
PREFERRED RESPONSE: 1
DISCUSSION: It is common for patients with ankylosing spondylitis to sustain extension-type fractures, typically near the cervicothoracic junction. These fractures can be minimally displaced, making them difficult to diagnose. In addition, the vertebral bodies are vascular and their canals are relatively enclosed, making them vulnerable to epidural bleeding. The MRI scan shows an epidural hematoma posteriorly compressing the cord.
REFERENCES: Bohlman HH: Acute fractures and dislocations of the cervical spine. J Bone Joint Surg Am 1979;61:1119-1142.
Weinstein PR, Karpman RR, Gall EP, et al: Spinal cord injury, spine fracture and spinal stenosis in ankylosing spondylitis. J Neurosurg 1982;57:609-616.
Johnson T, Steinbach L (eds): Essentials of Musculoskeletal Imaging. Rosemont, IL, American Academy of Orthopaedic Surgeons, 2003, p 44.
2. What are the most common portals for arthroscopic surgery of the ankle?
1- Anterolateral, anteromedial, posterolateral
2- Anterocentral, trans-Achilles, posterolateral
3- Anterocentral, anterolateral, posteromedial
4- Anterocentral, anteromedial, trans-Achilles
5- Anteromedial, anterolateral, trans-Achilles
PREFERRED RESPONSE: 1
DISCUSSION: The most commonly used portals are the anterolateral, anteromedial, and posterolateral portals. They have been shown to be the safest areas for portal placement, allowing no penetration of neurovascular structures. All the other portals involve placing another structure at risk. The anterocentral portal is close to the deep peroneal nerve and anterior tibular artery. The trans-Achilles portal is not recommended because of its limited utility and potential to injure the Achilles tendon. The posteromedial portal is too close to the posterotibial artery and nerve, the flexor hallucis longus and flexor digitorum longus tendons, and the branches of the calcaneal nerve.
REFERENCES: Stetson WB, Ferkel RD: Ankle arthroscopy: I. Technique and complications.
J Am Acad Orthop Surg 1996;4:17-23.
Ferkel RD: Diagnostic arthroscopic examination, in Ferkel RD, Whipple TL (eds): Arthroscopic Surgery: The Foot and Ankle. Philadelphia, PA, Lippincott-Raven, 1996, pp 103-118.
3. A patient who underwent primary total hip arthroplasty 7 years ago that resulted in excellent pain relief and a normal gait now reports pain and a limp. Postoperative and current AP radiographs are shown in Figures 2a and 2b. What is the most likely cause of the pathology seen?
1- Infection
2- Injury of the superior gluteal nerve
3- Loosening of the acetabular component
4- Loosening of the femoral component
5- Osteolysis
PREFERRED RESPONSE: 5
DISCUSSION: Osteolysis in the trochanteric bed can result in weakening of the bone and fracture. Nonsurgical management will provide reasonable clinical and radiographic results in patients with limited fracture displacement.
REFERENCES: Claus MC, Hopper RH, Engh CA: Fractures of the greater trochanter induced by osteolysis with the anatomic medullary locking prosthesis. J Arthroplasty 2002;17:706-712.
Heekin RD, Engh CA, Herzwurm MF: Fractures through cystic lesions of the greater trochanter: A cause of late pain after cementless total hip arthroplasty. J Arthroplasty 1996;11:757-760.
4. The safest surgical approach to the insertion of the tibial posterior cruciate ligament uses the interval between which of the following muscles?
1- Semimembranosus and medial gastrocnemius
2- Semimembranosus and semitendinosus
3- Medial and lateral gastrocnemius
4- Lateral gastrocnemius and biceps femoris tendon
5- Popliteus and lateral gastrocnemius
PREFERRED RESPONSE: 1
DISCUSSION: Burks and Schaffer described an approach to the tibial insertion of the posterior cruciate ligament that uses the interval between the semimembranosus and the medial gastrocnemius. The medial gastrocnemius muscle is retracted laterally and protects the neurovascular bundle. This approach is used to repair an avulsion of the posterior cruciate ligament tibial attachment or for performing a posterior cruciate ligament tibial
inlay reconstruction.
REFERENCES: Berg EE: Posterior cruciate ligament tibial inlay reconstruction. Arthroscopy 1995;8:95-99.
Burks RT, Schaffer JJ: A simplified approach to the tibial attachment of the posterior cruciate ligament. Clin Orthop 1990;254:216-219.
5. A 50-year-old man with no history of trauma reports new-onset back pain after doing some yard work the previous day. He reports pain radiating down his leg posteriorly and into the first dorsal web space of his foot. MRI scans are shown in Figures 3a through 3c. What nerve root is affected?
1- Left L4
2- Right L4
3- Left L5
4- Right L5
5- Left S1
PREFERRED RESPONSE: 3
DISCUSSION: The MRI scans clearly show an extruded L4-5 disk that is affecting the L5 nerve root on the left side. In addition, the L5 nerve root has a cutaneous distribution in the first dorsal web space. S1 affects the lateral foot. L4 affects the medial calf.
REFERENCE: An HS: Principles and Techniques of Spine Surgery. Baltimore, MD,
Williams and Wilkins, 1998, pp 98-100.
6. A 19-year-old wrestler has numbness along the radial aspect of the forearm after undergoing an open Bankart repair through an anterior deltopectoral approach. Motor weakness would be expected along with what other finding?
1- Diminished elbow flexion and supination strength
2- Reduced grip strength
3- Weakness in shoulder abduction
4- Weakness in flexion of the distal interphalangeal joint in the index finger
5- Weakness of the abductor digiti minimi
PREFERRED RESPONSE: 1
DISCUSSION: The musculocutaneous nerve may be injured by retracting the conjoined tendon medially. This nerve enters the coracobrachialis 5 cm distal to its origin. Its sensory distribution is the radial forearm, and its motor supply is to the biceps and brachialis.
REFERENCES: Bach BR, O’Brien SJ, Warren RF, et al: An unusual neurologic complication of the Bristow procedure. J Bone Joint Surg Am 1988;70:458-460.
McIlveen SJ, Duralde XA: Isolated nerve injuries about the shoulder, in Bigliani LU (ed): Complications of Shoulder Surgery. Baltimore, MD, Williams and Wilkins, 1993, pp 214-239.
7. A 19-year-old man has had intermittent progressive knee pain with ambulation and pain at night following a rodeo accident 4 weeks ago. Figures 4a through 4e show the radiographs, a bone scan, CT scan, and T2-weighted MRI scan. What is the most
likely diagnosis?
1- Infection
2- Osteosarcoma
3- Osteoblastoma
4- Giant cell tumor
5- Distal femoral fracture
PREFERRED RESPONSE: 2
DISCUSSION: The imaging studies reveal a predominantly blastic lesion in the distal femur with posterolateral periosteal changes. The bone scan shows increased uptake in the distal femur, beyond that expected with radiography. Cross-sectional imaging confirms the presence of a soft-tissue mass extending from the lateral aspect of the femur, with diffuse intramedullary signal changes. This aggressive presentation, particularly in this location and in a patient of this age, is most consistent with osteosarcoma. The mineralization in the soft tissue strongly suggests neoplasm, not the reactive bony changes seen in an infectious process.
REFERENCES: Menendez LR (ed): Orthopaedic Knowledge Update: Musculoskeletal Tumors. Rosemont, IL, American Academy of Orthopaedic Surgeons, 2002, pp 175-186.
Resnick D, Kyriakos M, Greenway GD: Tumors and tumor-like lesions of bone: Imaging and pathology of specific lesions, in Resnick D (ed): Diagnosis of Bone and Joint Disorders, ed 4. Philadelphia, PA, WB Saunders, 2002, vol 4, pp 3800-3833.
8. Figures 5a and 5b show the radiographs of an active 52-year-old man who has increasing knee pain and progressive varus deformity after undergoing total knee arthroplasty 7 years ago. Examination reveals a small effusion, but he has good motion and stability. What is the most likely diagnosis?
1- Wear-induced osteolysis
2- Giant cell tumor
3- Loose tibial component
4- Infection
5- Electrolytic reaction caused by dissimilar metals
PREFERRED RESPONSE: 1
DISCUSSION: The radiographs show narrowing of the medial joint space, which indicates polyethylene wear and progressive varus alignment. Wear particles incite osteolytic lesions like the one seen on the lateral radiograph.
REFERENCES: O’Rourke MR, Callaghan JJ, Goetz DG, et al: Osteolysis associated with a cemented modular posterior-cruciate-substituting total knee design. J Bone Joint Surg Am 2002;84:1362-1371.
Koval KJ (ed): Orthopaedic Knowledge Update 7. Rosemont, IL, American Academy of Orthopaedic Surgeons, 2002, pp 529-530.
9. Which of the following best describes the course of the median nerve at the elbow?
1- Crosses superficial to the ulnar artery, deep to the fibrous arch of the superficialis muscle, deep to the superficial head of the pronator teres muscle
2- Crosses superficial to the ulnar artery, deep to the fibrous arch of the superficialis muscle, superficial to the superficial head of the pronator teres muscle
3- Crosses deep to the ulnar artery, deep to the fibrous arch of the superficialis muscle, deep to the superficial head of the pronator teres muscle
4- Crosses deep to the ulnar artery, deep to the fibrous arch of the superficialis muscle, superficial to the superficial head of the pronator teres muscle
5- Crosses deep to the ulnar artery, superficial to the fibrous arch of the superficialis muscle, superficial to the superficial head of the pronator teres muscle
PREFERRED RESPONSE: 1
DISCUSSION: The median nerve courses superficial to the ulnar artery, deep to the fibrous arch of the superficialis muscle, and deep to the superficial head of the pronator teres muscle. The median nerve lies within the interval between the flexor digitorum superficialis muscle and the flexor digitorium muscle as it progresses toward the wrist.
REFERENCES: Hoppenfeld S, deBoer P: Surgical Exposures in Orthopaedics, ed 2. Philadelphia, PA, Lippincott-Raven, 1994, pp 118-131.
Netter F: The Ciba Collection of Medical Illustrations: The Musculoskeletal System. Part 1, Anatomy, Physiology and Metabolic Disorders. West Caldwell, NJ, Ciba-Geigy, 1991, vol 8,
pp 46-47.
10. A 62-year-old woman with soft-tissue calcifications and telangiectasia has severe pain in the left index, middle, ring, and little fingers. History reveals that she does not smoke. The clinical history and arteriogram shown in Figure 6 are consistent with which of the following conditions?
1- Buerger’s disease
2- Hypothenar hammer syndrome
3- Superficial palmar arch aneurysm
4- Raynaud’s phenomenon
5- Diabetic vasculopathy
PREFERRED RESPONSE: 4
DISCUSSION: The arteriogram shows generalized disease of all vascular structures. Even though the image was obtained following an infusion of nitroglycerin, little flow is present to the fingers. Based on the history of soft-tissue calcifications and telangiectasia, the most likely diagnosis is CREST (chondrocalcinosis, Raynaud’s phenomenon, esophageal dysmotility, sclerodactyly and telangiectasias). The arteriogram reveals Raynaud’s phenomenon or the “R” component of CREST. Buerger’s disease, or thromboangiitis obliterans, is strongly associated with a history of smoking. Hypothenar hammer syndrome involves repetitive trauma to the ulnar artery at the wrist, resulting in well-defined filling defects in the superficial palmar arch of the hand. Although not well visualized in this patient, the superficial arch is narrowed, showing no evidence of aneurysmal dilation.
REFERENCES: Koman LA, Ruch DS, Patterson Smith B, et al: Vascular disorders, in Green DP, Hotchkiss RN, Pederson WC (eds): Green’s Operative Hand Surgery, ed 4. New York, NY, Churchill Livingstone, 1999, vol 2, pp 2254-2302.
Miller LM, Morgan RF: Vasospastic disorders: Etiology, recognition, and treatment. Hand Clin 1993;9:171-187.
11. During excision of a Baker cyst, the base or stalk is usually found between the
1- popliteus muscle and posterior cruciate ligament.
2- posterior cruciate ligament and lateral gastrocnemius muscle.
3- medial gastrocnemius muscle and posterior cruciate ligament.
4- semimembranosus and medial head of the gastrocnemius muscles.
5- semitendinosus and medial head of the gastrocnemius muscles.
PREFERRED RESPONSE: 4
DISCUSSION: Although there are several bursae in the posterior portion of the knee, the most prevalent one with a connection to the knee joint is the one in the interval between the semimembranosus and the medial head of the gastrocnemius muscle. The popliteus muscle and posterior cruciate ligament, the posterior cruciate ligament and lateral gastrocnemius muscle, and the medial gastrocnemius muscle and posterior cruciate ligament are all too lateral and uncommon. The semitendinosus and medial head of the gastrocnemius muscles do not come in contact in the posterior aspect of the knee.
REFERENCES: Resnick D: Diagnosis of Bone and Joint Disorders, ed 3. Philadelphia, PA,
WB Saunders, 1995, p 379.
Justis EJ Jr: Nontraumatic disorders, in Crenshaw AH (ed): Campbell’s Operative Orthopaedics, ed 7. Philadelphia, PA, Lippincott, 1987, vol 3, p 2257.
12. A direct lateral (Hardinge) approach is used during total hip arthroplasty. The structure labeled A in Figure 7 is the
1- superior gluteal nerve.
2- inferior gluteal nerve.
3- obturator nerve.
4- medial femoral circumflex artery.
5- lateral femoral circumflex artery.
PREFERRED RESPONSE: 1
DISCUSSION: The superior gluteal nerve is located approximately 7.82 cm above the tip of the greater trochanter as it courses through the gluteus medius. This anatomic consideration is relevant during a Hardinge approach to the hip, where excessive proximal dissection or retraction could result in nerve injury. A split of the gluteus medius of no more than 4 cm above the greater trochanter is considered safe.
REFERENCES: Hoppenfeld S, deBoer P: Surgical Exposures in Orthopaedics: The Anatomic Approach. Philadelphia, PA, JB Lippincott, 1984, pp 333-335.
Ramesh M, O’Byrne JM, McCarthy N, et al: Damage to the superior gluteal nerve after the Hardinge approach to the hip. J Bone Joint Surg Br 1996;78:903-906.
13. The dorsal digital cutaneous nerve of the great toe shown in Figure 8 is a branch of
what nerve?
1- Saphenous
2- Medial branch of the superficial peroneal
3- Deep peroneal
4- Posterior tibial
5- Sural
PREFERRED RESPONSE: 2
DISCUSSION: The dorsal digital cutaneous nerve of the great toe is a branch of the medial branch of the superficial peroneal nerve. The deep peroneal nerve supplies the first web space.
REFERENCES: McMinn RMH, Hutchings RT, Logan BM: Color Atlas of Foot and Ankle Anatomy. Weert, Netherlands, Wolfe Medical Publications, 1982, p 50.
Gray H: Anatomy of the Human Body. Philadelphia, PA, Lea & Febiger, 2000, pp 963, 966.
14. In hip arthroplasty, the location of the medial femoral circumflex artery is best
described as
1- superior to the piriformis tendon.
2- superior to the anterior rim of the acetabulum.
3- deep to the transverse acetabular ligament.
4- deep to the quadratus femoris muscle.
5- within the substance of the gluteus minimus.
PREFERRED RESPONSE: 4
DISCUSSION: The obturator artery lies closest to the transverse acetabular ligament. The femoral artery is closest to the anterior rim of the acetabulum. No named vessel lies within the substance of the gluteus minimus or superior to the piriformis tendon. The medial femoral circumflex artery lies medial or deep to the quadratus femoris muscle.
REFERENCES: Hoppenfeld S, deBoer P: Surgical Exposures in Orthopaedics: The Anatomic Approach, ed 1. Philadelphia, PA, JB Lippincott, 1984, Figure 7-53, p 346.
Callaghan JJ, Rosenberg AG, Rubash HE: The Adult Hip. Philadelphia, PA, Lippincott-Raven, 1998, p 705.
15. A 47-year-old man has acute right shoulder pain after falling off a ladder. The MRI scan shown in Figure 9 reveals
1- aseptic necrosis of the humeral head.
2- a nondisplaced anatomic neck fracture.
3- a partial-thickness rotator cuff tear.
4- a full-thickness rotator cuff tear.
5- a bony Bankart defect.
PREFERRED RESPONSE: 4
DISCUSSION: The MRI scan reveals a full-thickness rotator cuff tear with retraction and increased signal in the subacromial space indicating joint fluid.
REFERENCES: Herzog RJ: Magnetic resonance imaging of the shoulder. Instr Course Lect 1998;47:3-20.
Iannotti JP, Zlatkin MB, Esterhai JL, et al: Magnetic resonance imaging of the shoulder: Sensitivity, specificity, and predictive value. J Bone Joint Surg Am 1991;73:17-29.
16. The oblique radiograph of the foot and the CT scan shown in Figures 10a and 10b show a patient whose symptoms have failed to respond to rest and non-steroidal anti-inflammatory drugs. What is the best course of action?
1- Excision of the damaged portion of the peroneus longus with possible transfer of the proximal peroneus longus into the peroneus brevis
2- Excision of the damaged portion of the peroneus brevis with possible transfer of the peroneus brevis into the peroneus longus
3- Excision of the bony fragments of the calcaneus with planing down of the lateral wall of the calcaneus to avoid irritation of the peroneal tendons
4- Casting to allow the avulsed portion of the base of the fifth metatarsal to heal
5- Debridement of the posterior tibial tendon, transfer of the flexor digitorum longus tendon into the navicular, and medial translational osteotomy of the calcaneus
PREFERRED RESPONSE: 1
DISCUSSION: The radiograph and MRI scan show elongation and fragmentation of the os peroneum. Although casting, orthoses, and steroid injection may relieve symptoms, excision of the os peroneum and primary repair when necessary, with or without tenodesis of the peroneus longus to the peroneus brevis, have been shown to produce excellent results.
REFERENCES: Haddad SL: Disorders of tendons: Peroneal tendon dysfunction, in Coughlin MJ, Mann RA (eds): Surgery of the Foot and Ankle, ed 7. St Louis, MO, Mosby, 1999,
pp 812-817.
MacDonald BD, Wertheimer SJ: Bilateral os peroneum fractures: Comparison of conservative and surgical treatment and outcomes. J Foot Ankle Surg 1997;36:220-225.
17. What is the typical MRI signal intensity of bone marrow affected by acute osteomyelitis?
1- Decreased on T1-weighted imaging, increased on T1-weighted imaging with gadolinium enhancement, increased on T2-weighted imaging
2- Decreased on T1-weighted imaging, increased on T1-weighted imaging with gadolinium enhancement, decreased on T2-weighted imaging
3- Increased on T1-weighted imaging, increased on T1-weighted imaging with gadolinium enhancement, increased on T2-weighted imaging
4- Increased on T1-weighted imaging, increased on T1-weighted imaging with gadolinium enhancement, decreased on T2-weighted imaging
5- Increased on T1-weighted imaging, decreased on T1-weighted imaging with gadolinium enhancement, decreased on T2-weighted imaging
PREFERRED RESPONSE: 1
DISCUSSION: The classic MRI findings of osteomyelitis are a decrease in the normally high signal intensity of marrow on T1-weighted images and normal or increased signal intensity on T2-weighted images. This is the result of replacement of marrow fat by inflammatory cells and edema, which causes lower signal intensity than fat on T1-weighted images and higher signal intensity than fat on T2-weighted images. The addition of gadolinium to a T1-weighted sequence reveals increased signal intensity in the hyperemic marrow.
REFERENCES: Unger E, Moldofsky P, Gatenby R, et al: Diagnosis of osteomyelitis by MR imaging. Am J Roentgenol 1988;150:605-610.
Dormans JP, Drummond DS: Pediatric hematogenous osteomyelitis: New trends in presentation, diagnosis and treatment. J Am Acad Orthop Surg 1994;2:333-341.
Herring JA: Tachdjian’s Pediatric Orthopaedics, ed 3. Philadelphia, PA, WB Saunders, 2002, vol 1, pp 150-163.
18. Based on the appearance of the imaging studies shown in Figures 11a through 11c, what structure has most likely been injured?
1- Quadriceps tendon
2- Medial collateral ligament
3- Medial patellofemoral ligament
4- Anterior cruciate ligament
5- Lateral retinaculum
PREFERRED RESPONSE: 3
DISCUSSION: The radiographs reveal marked lateral subluxation of the patella in a patient who has recurrent patellar instability. The medial patellofemoral ligament is the main restraint to lateral subluxation of the patella.
REFERENCES: Boden BP, Pearsall AW: Patellofemoral instability: Evaluation and management. J Am Acad Orthop Surg 1997;5:47-57.
Desio SM, Burks RT, Bachus KN: Soft tissue restraints to lateral patellar translation in the human knee. Am J Sports Med 1998;26:59-65.
19. In the anterior forearm approach to the distal radius (Henry approach), the radial artery is located between what two structures?
1- Flexor carpi radialis tendon and flexor digitorum superficialis muscle
2- Flexor carpi radialis and brachioradialis tendons
3- Flexor carpi radialis and palmaris longus tendons
4- Brachioradialis and flexor pollicis longus tendons
5- Brachioradialis tendon and flexor digitorum superficialis muscle
PREFERRED RESPONSE: 2
DISCUSSION: The standard approach to the volar aspect of the distal radius is the Henry approach. Following incision of the skin and subcutaneous tissues, the forearm fascia is incised. The radial artery and venae comitantes lie in the interval between the tendons of the flexor carpi radialis muscle and the brachioradialis muscle. This interval is developed, and the radial artery and veins are retracted in a radial direction.
REFERENCES: Hoppenfeld S, deBoer P: Surgical Exposures in Orthopaedics, ed 2. Philadelphia, PA, Lippincott-Raven, 1994, pp 118-131.
Henry A: Extensile Exposure, ed 3. Edinburgh, UK, Churchill Livingstone, 1995, pp 100-107.
20. Following a radial nerve neurapraxia at or above the elbow, return of muscle function
can be expected to start at the brachioradialis and return along which of the
following progressions?
1- Extensor carpi radialis brevis, extensor carpi radialis longus, extensor digitorum comminus, extensor indicis proprious
2- Extensor carpi radialis brevis, extensor carpi radialis longus, extensor indicis proprious, extensor digitorum comminus
3- Extensor carpi radialis longus, extensor carpi radialis brevis, extensor indicis proprious, extensor digitorum comminus
4- Extensor carpi radialis longus, extensor carpi radialis brevis, extensor digitorum comminus, extensor indicis proprious
5- Extensor carpi radialis longus, extensor digitorum comminus, extensor carpi radialis brevis, extensor indicis proprious
PREFERRED RESPONSE: 4
DISCUSSION: Following a radial nerve neurapraxia above the elbow, muscle recovery can be expected in a predictable pattern. Although variations will occur, the return of function or reinnervation usually occurs in the following order: brachioradialis, extensor carpi radialis longus, supinator, extensor carpi radialis brevis, extensor carpi ulnaris, extensor digitorum comminus, extensor digiti minimi, extensor indicis proprious, extensor pollicis longus, abductor pollicis longus, and extensor pollicis brevis.
REFERENCES: Netter F: The Ciba Collection of Medical Illustrations: The Musculoskeletal System. Part 1, Anatomy, Physiology and Metabolic Disorders. West Caldwell, NJ,
Ciba-Geigy, 1991, vol 8, p 53.
Hollinshead WH: Anatomy for Surgeons: The Back and Limbs, ed 3. Philadelphia, PA,
Harper and Row, 1982, vol 3, pp 428-429.
21. To preserve blood supply to the fractured bone seen in Figures 12a and 12b, care should be taken when exposing which of the following areas?
1- Anterolateral aspect of the distal humerus
2- Posterolateral aspect of the distal humerus
3- Medial epicondyle
4- Lateral epicondyle
5- Radial neck
PREFERRED RESPONSE: 2
DISCUSSION: The blood supply to the adult capitellum and lateral trochlea comes from posterior vessels arising from the radial recurrent, radial collateral, and interosseous recurrent arteries. These arteries penetrate the distal humerus posterior and superior to the capitellum.
REFERENCE: Yamaguchi K, Sweet FA, Bindra R, et al: The extraosseous and intraosseous arterial anatomy of the adult elbow. J Bone Joint Surg Am 1997;79:1653-1662.
22. An axial T1-weighted MRI scan of the pelvis is shown in Figure 13. The arrow is pointing to what muscle?
1- Adductor magnus
2- Pectineus
3- Obturator externus
4- Obturator internus
5- Levator ani
PREFERRED RESPONSE: 4
DISCUSSION: The obturator internus muscle originates from the internal pelvic wall and passes laterally through the lesser sciatic foramen, banking around the ischium below the sacrospinous ligament before inserting on the medial aspect of the greater trochanter.
REFERENCES: Higuchi T: Normal anatomy and magnetic resonance appearance of the pelvis, in Takahashi HE, Morita T, Hotta T, Ogose A (eds): Operative Treatment of Pelvic Tumors. Tokyo, Japan, Springer-Verlag, 2003, pp 4-21.
Berquist TH: Pelvis, hips and thigh, in Berquist TH (ed): MRI of the Musculoskeletal System,
ed 4. Philadelphia, PA, Lippincott Williams and Wilkins, 2001, pp 210-238.
23. Which of the following radiographic views best depicts a Hill-Sachs defect?
1- Outlet
2- True AP
3- Serendipity
4- Stryker notch
5- Zanca
PREFERRED RESPONSE: 4
DISCUSSION: The Stryker notch view best shows this type of defect. An outlet view helps evaluate acromial shape, a true AP shows joint space narrowing, a serendipity view evaluates the sternoclavicular joint, and a Zanca view helps evaluate the acromioclavicular joint. An internal rotation AP may also depict a Hill-Sachs defect.
REFERENCE: Rockwood CA, et al: X-ray evaluation of shoulder problems, in Rockwood CA, Matsen FA (eds): The Shoulder. Philadelphia, PA, WB Saunders, 1990, vol 1, pp 178-207.
24. What structure provides the major blood supply to the humeral head?
1- Posterior circumflex humeral artery
2- Anterior circumflex humeral artery, ascending branch
3- Nutrient humeral artery
4- Thoracoacromial artery, deltoid branch
5- Small arteries of the rotator cuff insertions
PREFERRED RESPONSE: 2
DISCUSSION: The ascending branch of the anterior circumflex humeral artery provides
the major blood supply to the humeral head. The posterior circumflex humeral artery
supplies a much smaller portion of the proximal humerus. The nutrient humeral artery is the main blood supply for the humeral shaft. The thoracoacromial artery is primarily a muscular branch. The rotator cuff insertions contribute some blood supply to the tuberosities but not a major contribution.
REFERENCES: Beaty JH (ed): Orthopaedic Knowledge Update 6. Rosemont, IL, American Academy of Orthopaedic Surgeons, 1999, pp 271-286.
Cushner MA, Friedman RJ: Osteonecrosis of the humeral head. J Am Acad Orthop Surg 1997;5:339-346.
25. Figure 14 shows an intra-articular gadolinium-enhanced MRI scan of a 52-year-old woman who has stopped playing tennis because of pain in her left shoulder while serving. What is the most likely diagnosis?
1- Superior labral detachment
2- Os acromiale
3- Partial-thickness rotator cuff tear on the articular side
4- Medial subluxation of the long head of the biceps
5- Moderate arthrosis of the glenohumeral joint
PREFERRED RESPONSE: 3
DISCUSSION: The MRI scan shows increased signal intensity along the deep fibers of the supraspinatus near its insertion. This is typical of tendinosis and a probable partial-thickness rotator cuff tear.
REFERENCES: Herzog RJ: Magnetic resonance imaging of the shoulder. Instr Course Lect 1998;47:3-20.
Iannotti JP, Zlatkin MB, Esterhai JL, et al: Magnetic resonance imaging of the shoulder: Sensitivity, specificity, and predictive value. J Bone Joint Surg Am 1991;73:17-29.
26. To adequately expose the volar plate of the proximal interphalangeal joint of the finger, which of following pulleys is typically incised?
1- Entire A3, entire C2, and the proximal portion of A4
2- Entire C1 only
3- Entire C2 only
4- Distal portion of A2, entire C1, and entire A3
5- Distal portion of C1, entire A3, and the proximal portion of C2
PREFERRED RESPONSE: 5
DISCUSSION: Full exposure of the volar plate of the proximal interphalangeal joint of the finger is best accomplished by incision of the distal C1, A3, and proximal C2 pulleys; followed by gentle retraction of the flexor digitorum superficialis and profundus tendons. Sacrifice of the A3 pulley, although associated with some biomechanic disadvantage, can be tolerated without causing functionally limiting bowstringing of the flexor tendon. Sacrifice of even a portion of the A2 or A4 pulleys can decrease the biomechanic leverage provided by the flexor tendon sheath, leading to bowstringing of the flexor tendons.
REFERENCES: Hoppenfeld S, deBoer P: Surgical Exposures in Orthopaedics, ed 2. Philadelphia, PA, Lippincott-Raven, 1994, pp 176-186.
Strickland J: Flexor tendon-acute injuries, in Green DP, Hotchkiss RN, Pederson WC (eds): Green’s Operative Hand Surgery, ed 4. New York, NY, Churchill Livingstone, 1999, vol 2,
pp 1853-1855.
Lin GT, Amadio PC, An KN, et al: Functional anatomy of the human digital flexor pulley system. J Hand Surg Am 1989;14:949-956.
27. A 42-year-old patient has had a fever and low back pain for several days. Laboratory studies show an elevated erythrocyte sedimentation rate and a WBC count of 9,500 mm3 with 75% neutrophils. A CT scan is shown in Figure 15. Examination will most likely reveal what other findings?
1- Cauda equina syndrome
2- Pain improved with hip flexion
3- Pain in the lateral aspect of the thigh
4- Pain in the sole of the foot
5- Pain in the upper thoracic spine
PREFERRED RESPONSE: 2
DISCUSSION: The CT scan reveals a left-sided psoas abscess. Irritation of the saphenous division of the femoral nerve can cause paresthesias along the medial aspect of the knee. Pain is usually improved with hip flexion.
REFERENCES: Cellier C, Gendre JP, Cosnes J, et al: Psoas abscess complication Crohn’s disease. Gastroenterol Clin Biol 1992;16:235-238.
Netter FH: Atlas of Human Anatomy. Summit, NJ, Ciba-Geigy, 1989, pp 470-471, 506.
28. Based on the diagram shown in Figure 16, what muscle derives its innervation from the nerve identified by the letter “A”?
1- Pectoralis minor
2- Teres minor
3- Subclavius
4- Brachialis
5- Supraspinatus
PREFERRED RESPONSE: 2
DISCUSSION: The nerve labeled A is the axillary nerve, a branch from the posterior cord. The posterior cord innervates the subscapularis, latissimus dorsi, teres major and minor, deltoid, triceps, anconeus, brachioradialis, and extensors of the forearm. The axillary nerve innervates the teres minor and deltoid. The pectoralis minor is innervated by the medial cord. The supraspinatus and the subclavius are innervated by the superior trunk. The brachialis is innervated by the lateral cord.
REFERENCES: Moore K: Anatomy, ed 3. Philadelphia, PA, Williams and Wilkins, 1992.
Netter FH: Atlas of Human Anatomy. Summit, NJ, Ciba-Geigy, 1989, pp 400, 405, 407, 450.
29. In performing an opening wedge high tibial osteotomy at the tibial tubercle, the osteotome extends 5 mm posteriorly and centrally out of the bone as shown in Figures 17a and 17b. What is the first structure it enters?
1- Popliteal artery
2- Popliteal vein
3- Tibial nerve
4- Popliteus muscle
5- Soleus muscle
PREFERRED RESPONSE: 4
DISCUSSION: The major risk of performing a high tibial osteotomy is neurovascular injury. The new version of the high tibial osteotomy makes a transverse osteotomy at the level of the tibial tubercle. The osteotome is protected by the oblique belly of the popliteus muscle. The popliteal artery and vein and tibial nerve all lie posterior to the muscle. The soleus muscle originates below this level.
REFERENCES: Clement CD: Anatomy: A Regional Atlas of Human Anatomy, ed 3.
Baltimore, MD, Munich, Germany, Urban and Schwarzberg, 1987, Figure 422.
Netter FH: Atlas of Human Anatomy. Summit, NJ, Ciba-Geigy, 1989, plate 480.
30. The arrow in the axial T1-weighted MRI scan shown in Figure 18 is pointing to which of the following structures?
1- Ulnar artery
2- Ulnar nerve in Guyon’s canal
3- Deep branch of the ulnar nerve only
4- Median nerve
5- Radial artery
PREFERRED RESPONSE: 2
DISCUSSION: The arrow is pointing to the ulnar nerve within Guyon’s canal. Guyon’s canal is approximately 4 cm long, beginning at the proximal extent of the transverse carpal ligament and ending at the aponeurotic arch of the hypothenar muscles. Many structures comprise the boundaries of Guyon’s canal. The floor, for example, consists of the transverse carpal ligament, the pisohamate and pisometacarpal ligaments, and the opponens digiti minimi. Within Guyon’s canal, the ulnar nerve bifurcates into the superficial and deep branches, with the deep branch of the ulnar nerve persisting distal to the canal. The ulnar artery is immediately adjacent and radial to the ulnar nerve. The median nerve is visualized within the carpal tunnel. The radial artery is on the radial side of the wrist. The hook of the hamate is clearly seen in the figure, orienting the observer to the ulnar side of the wrist.
REFERENCES: Goss MS, Gelberman RH: The anatomy of the distal ulnar tunnel. Clin Orthop 1985;196:238-247.
Denman EE: The anatomy of the space of Guyon. Hand 1978;10:69-76.
31. Osteonecrosis of the femoral head after intramedullary nailing in children is thought to be the result of injury to the
1- artery within the ligamentum teres.
2- medial femoral circumflex artery.
3- lateral ascending cervical artery.
4- lateral femoral circumflex artery.
5- intraosseous blood supply.
PREFERRED RESPONSE: 3
DISCUSSION: All of these are possible explanations for the development of osteonecrosis following intramedullary nailing in children. However, the lateral ascending cervical artery, which supplies the epiphysis, is much more vulnerable to injury in children because it lies in the trochanteric fossa.
REFERENCES: Buckley SL: Current trends in the treatment of femoral shaft fractures in children and adolescents. Clin Orthop 1997;338:60-73.
Rockwood CA, Wilkins KE, Beaty JH: Fractures in Children, ed 4. Philadelphia, PA, Lippincott-Raven, 1996, p 1214.
32. The illustration shown in Figure 19 shows a Chamberlain line. What is the most
likely diagnosis?
1- Anterior atlanto-occipital dislocation
2- Basilar invagination
3- C1-C2 dislocation
4- Transverse ligament injury
5- Normal findings
PREFERRED RESPONSE: 5
DISCUSSION: Basilar invagination is best defined as vertical or compressive instability at the occiput-C1 joint. Such invaginations most commonly occur in patients with rheumatoid arthritis but also can occur secondary to trauma or tumor. A Chamberlain line is used as a method to determine basilar invagination. The odontoid tip should not be more than 5 mm above a Chamberlain line.
REFERENCES: Wiesel SW, Rothman RH: Occipito-atlantal hypermobility.
Spine 1979;4:187-191.
Clark CR: The Cervical Spine, ed 3. Philadelphia, PA, Lippincott-Raven, 1998, pp 50-51.
33. Figures 20a and 20b show the sagittal and coronal T1-weighted MRI scans of a patient’s left knee. Abnormal findings include
1- enchondroma of the proximal tibia.
2- a bone bruise of the lateral femoral condyle.
3- a tear of the lateral collateral ligament.
4- a tear of the discoid lateral meniscus.
5- a physeal fracture of the proximal femur.
PREFERRED RESPONSE: 4
DISCUSSION: The MRI scans show meniscal tissue extending across the entire lateral compartment, revealing a discoid lateral meniscus. The increased signal within the lateral meniscal tissue indicates a tear. Discoid lateral menisci are congenital variants that often present with mechanical symptoms in adolescents. The other structures in the knee are normal.
REFERENCES: Ahn JH, Shim JS, Hwang CH, et al: Discoid lateral meniscus in children: Clinical manifestations and morphology. J Pediatr Orthop 2001;21:812-816.
Andrish JT: Meniscal injuries in children and adolescents: Diagnosis and management.
J Am Acad Orthop Surg 1996;4:231-237.
34. An ulnar nerve palsy at the level of the wrist is typically associated with deficits in the palmaris brevis, the hypothenar muscles, and what other groups of muscles?
1- Volar interossei, adductor pollicis, and the deep head of the flexor pollicis brevis
2- Dorsal interossei, adductor pollicis, and the deep head of the flexor pollicis brevis
3- Dorsal interossei, adductor pollicis, and both heads of the flexor pollicis brevis
4- All of the interossei and the abductor pollicis brevis
5- All of the interossei, adductor pollicis, and the deep head of the flexor pollicis brevis
PREFERRED RESPONSE: 5
DISCUSSION: The intrinsic muscles innervated by the ulnar nerve include the palmaris brevis, hypothenar muscles, all of the interossei, adductor pollicis, and the deep head of the flexor pollicis brevis. The superficial head of the flexor pollicis brevis is innervated by the
median nerve.
REFERENCES: Goldfarb CA, Stern PJ: Low ulnar nerve palsy. JASSH 2003;3:14-26.
Omer G: Ulnar nerve palsy, in Green DP, Hotchkiss R, Pederson W (eds): Green’s Operative Hand Surgery, ed 4. Philadelphia, PA, Churchill Livingstone, 1999, pp 1526-1541.
35. Figures 21a and 21b show the radiographs of a 22-year-old man who has had progressive pain and swelling about the knee for the past 6 weeks. Examination reveals limited range of motion and fullness about the knee. What is the most likely diagnosis?
1- Giant cell tumor
2- Aneurysmal bone cyst
3- Osteoblastoma
4- Osteosarcoma
5- Metastatic carcinoma
PREFERRED RESPONSE: 4
DISCUSSION: The radiographs reveal a destructive lesion in the metaphysis of the distal femur with periosteal changes and an associated soft-tissue mass with subtle mineralization. This suggests an aggressive malignant process. In this age group, the most likely diagnosis is osteosarcoma. Giant cell tumor, which usually is in a more subchondral location, is not typically so aggressive. Aneurysmal bone cyst is usually more geographic, with a well-marginated reactive rim.
REFERENCES: Menendez LR (ed): Orthopaedic Knowledge Update: Musculoskeletal Tumors. Rosemont, IL, American Academy of Orthopaedic Surgeons, 2002, pp 175-186.
Resnick D, Kyriakos M, Greenway GD: Tumors and tumor-like lesions of bone: Imaging and pathology of specific lesions, in Resnick D (ed): Diagnosis of Bone and Joint Disorders, ed 4. Philadelphia, PA, WB Saunders, 2002, vol 4, pp 3800-3833.
36. The anterolateral (Watson-Jones) approach to the hip exploits the intermuscular interval between the
1- gluteus medius and tensor fascia lata.
2- gluteus medius and minimus.
3- gluteus medius and maximus.
4- gluteus minimus and piriformis.
5- tensor fascia lata and sartorius.
PREFERRED RESPONSE: 1
DISCUSSION: The Watson-Jones approach to the hip uses the intermuscular interval between the gluteus medius and the tensor fascia lata. This is not a true internervous plane, as both muscles are supplied by the superior gluteal nerve.
REFERENCES: Hoppenfeld S, deBoer P: Surgical Exposures in Orthopaedics: The Anatomic Approach. Philadelphia, PA, JB Lippincott, 1984, pp 316-332.
Crenshaw AH (ed): Campbell’s Operative Orthopedics, ed 7. St Louis, MO, CV Mosby, 1987,
p 63.
37. An 8-month-old infant has an infection of the fingertip as shown in Figure 22. If neglected, the anticipated path of ascending infection is the fingertip, the flexor sheath, and the
1- midpalmar space.
2- thenar space.
3- hypothenar space.
4- ulnar bursa.
5- radial bursa.
PREFERRED RESPONSE: 1
DISCUSSION: The flexor sheaths are in continuity with the deep spaces of the hand. The flexor sheaths of the thumb and little finger communicate with the radial and ulnar bursae, respectively, and these two bursae commonly communicate. The central digits do not communicate as readily with deep spaces of the hand but if flexor tendon sheath infection of the index, long, and right fingers is neglected, the potential exists for rupture into the deep midpalmar spaces.
REFERENCES: Peimer CA (ed): Surgery of the Hand and Upper Extremity: Acute and Chronic Sepsis. New York, NY, Mcgraw Hill, 1996, pp 1735-1741.
Trumble TE (ed): Hand Surgery Update 3: Hand, Elbow and Shoulder. Rosemont, IL, American Society for Surgery of the Hand, 2003, pp 445-446.
38. A 24-year-old man has had pain in the left knee for the past several months. He reports that initially the pain was associated with weight-bearing activities, but it has now become more constant. He denies any swelling but reports a lateral fullness at the tibial plateau. Figures 23a through 23e show radiographs, a bone scan, and T1- and T2-weighted MRI scans. What is the most likely diagnosis?
1- Stress fracture
2- Simple bone cyst
3- Fibrous dysplasia
4- Infection
5- Giant cell tumor
PREFERRED RESPONSE: 5
DISCUSSION: The radiographs reveal a lytic subchondral lesion that has a poorly defined margin and lacks mineralization. The bone scan confirms an active lesion that has central photopenia, producing the characteristic doughnut configuration. The MRI scans confirm the presence of a subchondral lesion that is modestly expansile at the lateral plateau and has low signal intensity on the T1-weighted image and a mixed high signal on the T2-weighted image. These features strongly suggest giant cell tumor of bone, more than 50% of which appear around the knee. Simple cyst is excluded by the MRI characteristics. Fibrous dysplasia is unlikely to be in a subchondral location and typically does not show this intensity of uptake on bone scan.
REFERENCES: Parsons TW: Benign bone tumors, in Fitzgerald RH, Kaufer H,
Malkani AL (eds): Orthopaedics. St Louis, MO, Mosby, 2002, pp 1027-1035.
Resnick D, Kyriakos M, Greenway GD: Tumors and tumor-like lesions of bone: Imaging and pathology of specific lesions, in Resnick D (ed): Diagnosis of Bone and Joint Disorders, ed 4. Philadelphia, PA, WB Saunders, 2002, vol 4, pp 3939-3962.
39. Figure 24 shows an axial MRI scan of the ankle. The arrowhead is pointing to
what structure?
1- Tibial nerve
2- Flexor hallucis longus
3- Tibialis posterior
4- Peroneus longus
5- Peroneus brevis
PREFERRED RESPONSE: 5
DISCUSSION: The peroneus brevis is easily identified by its location behind the fibula and its distal muscle belly. Axial MRI images provide a reliable guide even when one of the peroneals is completely ruptured, subluxated out of the peroneal groove, or absent.
REFERENCES: Sarrafian SK: Anatomy of the Foot and Ankle: Descriptive, Topographic, Functional, ed 2. New York, NY, Lippincott, 1993, pp 234-235.
Sammarco GJ: Peroneus longus tendon tears: Acute and chronic. Foot Ankle Int
1995;16:245-253.
40. During total hip arthroplasty, profuse bleeding is noted following predrilling for placement of an acetabular component screw. The drill most likely penetrated too deep in the
1- anterior-superior acetabular quadrant.
2- posterior-superior acetabular quadrant.
3- posterior-inferior acetabular quadrant.
4- medial acetabular wall.
5- ischial body.
PREFERRED RESPONSE: 1
DISCUSSION: The acetabular quadrants are defined by two lines: one drawn from the anterosuperior iliac spine to the posterior fovea, forming acetabular halves, and a second drawn perpendicular to the first at the midpoint of the acetabulum, forming four quadrants. The anterior quadrants should be avoided because improper screw placement may injure the external iliac artery and vein, as well as the obturator nerve, artery, and vein. These structures lie close to the pelvic bone, with little protective interposition of soft tissue.
REFERENCES: Wasielewski RC, Cooperstein LA, Kruger MP, et al: Acetabular anatomy and the transacetabular fixation of screws in total hip arthroplasty. J Bone Joint Surg Am 1990;72:501-508.
Keating EM, Ritter MA, Faris PM: Structures at risk from medially placed acetabular screws.
J Bone Joint Surg Am 1990;72:509-511.
41. A posterolateral approach to the tibial plafond proceeds between what two muscles?
1- Peroneus longus and brevis
2- Peroneus longus and flexor hallucis longus
3- Peroneus brevis and flexor hallucis longus
4- Peroneus brevis and flexor digitorum longus
5- Flexor hallucis longus and flexor digitorum longus
PREFERRED RESPONSE: 3
DISCUSSION: A posterolateral approach to the posterior malleolus proceeds between the
lateral and deep posterior compartments. Distally, the peroneus brevis muscle lies most
medially within the lateral compartment, and the flexor hallucis longus lies most laterally
in the deep posterior compartment.
REFERENCES: Henry AK: Extensile Exposure, ed 2. Edinburgh, UK, Churchill Livingstone, 1973, pp 269-270.
Hoppenfeld S, deBoer P: Surgical Exposures in Orthopaedics: The Anatomic Approach. Philadelphia, PA, JB Lippincott, 1984, p 515.
42. The brachialis muscle is innervated by what two nerves?
1- Radial and musculocutaneous
2- Lateral and medial brachial cutaneous
3- Axillary and medial and lateral pectoral
4- Radial and ulnar
5- Ulnar and musculocutaneous
PREFERRED RESPONSE: 1
DISCUSSION: The brachialis is innervated by two nerves: medially, the musculocutaneous nerve; laterally, the radial nerve. The muscle is split longitudinally to approach the humerus anteriorly.
REFERENCES: Henry AK: The distal part of the humerus and front of the forearm, in Henry AK (ed): Extensile Exposure, ed 2. Edinburgh, UK, Churchill Livingstone, 1973, pp 90-115.
King A, Johnston GH: A modification of Henry’s anterior approach to the humerus. J Shoulder Elbow Surg 1998;7:210-212.
43. Figure 25 shows the CT scan of an adult patient who has neck pain following a motor vehicle accident. What is the most likely diagnosis?
1- Jefferson’s fracture
2- C1-C2 rotational instability
3- Transverse ligament rupture
4- Normal finding
5- Basilar invagination
PREFERRED RESPONSE: 3
DISCUSSION: If the atlanto-dens interval is greater than 3 mm in an adult, a transverse ligament rupture usually is suspected. The atlanto-dens interval can be seen with CT or in lateral radiographs of the upper cervical spine. Transverse ligament rupture can occur as an isolated entity or in association with an odontoid or a Jefferson’s fracture. Patients with this type of injury usually require fusion.
REFERENCES: Dickman CA, Greene KA, Sonntag VK: Injuries involving the transverse atlantal ligament: Classification and treatment guidelines based upon experience with 39 injuries. Neurosurgery 1996;38:44-50.
Clark CR: The Cervical Spine, ed 3. Philadelphia, PA, Lippincott-Raven, 1998, pp 362-363.
44. Which of the following best describes the course of the ulnar nerve in the midforearm?
1- Travels deep to the flexor carpi ulnaris muscle, radial to the ulnar artery, superficial to the flexor digitorum profundus muscle
2- Travels deep to the flexor carpi ulnaris muscle, radial to the ulnar artery, deep to the flexor digitorum profundus muscle
3- Travels deep to the flexor carpi ulnaris muscle, ulnar to the ulnar artery, superficial to the flexor digitorum profundus muscle
4- Travels superficial to the flexor carpi ulnaris muscle, ulnar to the ulnar artery, superficial to the flexor digitorum profundus muscle
5- Travels superficial to the flexor carpi ulnaris muscle, radial to the ulnar artery, superficial to the flexor digitorum profundus muscle
PREFERRED RESPONSE: 3
DISCUSSION: In the midforearm, the ulnar nerve travels deep to the flexor carpi ulnaris muscle and ulnar to the ulnar artery as it lies on the flexor digitorum profundus muscle. In this region, the ulnar nerve and artery lie side-by-side, whereas more proximal in the forearm, the ulnar artery originates from the brachial artery in the antecubital fossa, and the ulnar nerve lies within the cubital tunnel.
REFERENCES: Hoppenfeld S, deBoer P: Surgical Exposures in Orthopaedics, ed 2. Philadelphia, PA, Lippincott-Raven, 1994, pp 118-131.
Netter F: The Ciba Collection of Medical Illustrations: The Musculoskeletal System: Part 1, Anatomy, Physiology and Metabolic Disorders. West Caldwell, NJ, Ciba-Geigy, 1991, vol 8,
pp 46-47.
45. A 70-year-old former baseball catcher reports long-standing pain in the ring and
little fingers. A gradient-echo MRI scan is shown in Figure 26. What is the most
likely diagnosis?
1- Giant cell tumor of the tendon sheath
2- Ulnar artery aneurysm
3- Lipoma
4- Hematoma
5- Neurolemmoma
PREFERRED RESPONSE: 2
DISCUSSION: The gradient-echo MRI scan highlights the ulnar and radial arteries,
as indicated by the arrow. This technique suppresses the signal of the surrounding fat
and causes the stationary surrounding tissues to become intermediate in signal intensity.
The flowing blood is then easily identified with a bright signal because it does not absorb the radiofrequency pulse. Based on the findings, the diagnosis is an ulnar artery aneurysm, most likely caused by years of repetitive trauma as the result of catching baseballs. Neurolemmoma and giant cell tumor of the tendon sheath would be intermediately enhanced on this image sequence, and the continuity with the ulnar artery, demonstrated here, would not be expected. Lipomas are not enhanced using the gradient-echo technique. The chronic nature of the patient’s symptoms is not indicative of a hematoma, and the hematoma would be dark on this imaging sequence since it is stationary tissue.
REFERENCES: Koman LA, Ruch DS, Patterson Smith B, et al: Vascular disorders, in Green DP, Hotchkiss RN, Pederson WC (eds): Green’s Operative Hand Surgery, ed 4. New York, NY, Churchill Livingstone, 1999, vol 2, pp 2254-2302.
Holder LE, Merine DS, Yang A: Nuclear medicine, contrast angiography, and magnetic resonance imaging for evaluating vascular problems in the Hand: Vasospastic disorders. Hand Clin 1993;9:95-113.
46. In a postganglionic brachial plexus lesion at Erb’s point (point of formation of the
upper trunk by the C5 and C6 nerve roots), which of the following nerves will still function normally?
1- Suprascapular
2- Musculocutaneous
3- Axillary
4- Lateral pectoral
5- Dorsal scapular
PREFERRED RESPONSE: 5
DISCUSSION: In a postganglionic injury to the brachial plexus, the rhomboid muscle, innervated by the dorsal scapular nerve, would still be expected to function. This is a useful clinical sign that the brachial plexus lesion is postganglionic as opposed to preganlionic. The musculocutaneous, axillary, and suprascapular nerves are all located distal to Erb’s point (the most common location of an upper nerve root brachial plexus injury), and all contain fibers from the C5 and C6 nerve roots. Therefore, these nerves are not expected to function normally following a postganglionic C5 and C6 nerve root injury.
REFERENCES: Netter F: The Ciba Collection of Medical Illustrations: The Musculoskeletal System: Part 1, Anatomy, Physiology and Metabolic Disorders. West Caldwell, NJ, Ciba-Geigy, 1991, vol 8, pp 28-29.
Zimmerman NB, Weiland AJ: Assessment and monitoring of brachial plexus injury in the adult, in Gelberman RH (ed): Operative Nerve Repair and Reconstruction. Philadelphia, PA,
JB Lippincott, 1991, vol 2, pp 1273-1283.
47. The posterior circumflex humeral artery and the axillary nerve usually lie in a space bordered superiorly by the
1- teres minor, medially by the long head of the triceps, laterally by the humerus, and inferiorly by the teres major.
2- teres major, medially by the humerus, and inferiorly by the teres minor.
3- supraspinatus, inferiorly by the infraspinatus, and posterior to the scapular body.
4- supraspinatus, inferiorly by the scapular body, and covered by the superior transverse scapular ligament.
5- infraspinatus, inferiorly by the teres minor, and laterally by the long head of the triceps.
PREFERRED RESPONSE: 1
DISCUSSION: The quadrangular space is bordered superiorly by the teres minor, medially by the long head of the triceps, laterally by the humerus, and inferiorly by the teres major. The posterior circumflex humeral artery and the axillary nerve lie in this space.
REFERENCES: Rockwood CA Jr, Matsen FA III: The Shoulder, ed 2. Philadelphia, PA,
WB Saunders, 1998, vol 1, pp 70-71.
Netter FH: Atlas of Human Anatomy. Summit, NJ, Ciba-Geigy, 1989, pp 401-402, 407.
48. A patient notes pain under the first metatarsophalangeal joint following a soccer injury. The MRI scans shown in Figures 27a and 27b reveal what pathologic finding?
1- Sesamoid fracture
2- Phalangeal fracture
3- Osteochondral lesion
4- Disruption of the plantar plate
5- Rupture of the flexor hallucis tendon
PREFERRED RESPONSE: 4
DISCUSSION: The MRI scans show a complete disruption of the sesamoid complex with proximal retraction of the medial sesamoid and high signal originating from the site normally occupied by the plantar plate (metatarsophalangeal ligament). This injury is the result of a hyperextension injury and is a severe variant of a turf toe.
REFERENCES: Watson TS, Anderson RB, Davis WH: Periarticular injuries to the hallux metatarsophalangeal joint in athletes. Foot Ankle Clin 2000;5:687-713.
Tewes DP, Fischer DA, Fritts HM, et al: MRI findings of acute turf toe: A case report and review of anatomy. Clin Orthop 1994;304:200-203.
49. When performing the exposure for an anterior approach to the cervical spine, excessive retraction of the trachea and esophagus should be avoided to prevent injury of the
1- vagus nerve.
2- recurrent laryngeal nerve.
3- superior laryngeal nerve.
4- hypoglossal nerve.
5- sympathetic trunk.
PREFERRED RESPONSE: 2
DISCUSSION: The recurrent laryngeal nerve lies between the trachea and the esophagus and is subject to stretch injury if excessive retraction is applied. The vagus nerve lies in the carotid sheath. The sympathetic trunk lies anterior to the longus colli muscles. The hypoglossal nerve and superior laryngeal nerve are both at risk during the exposure but are not located between the trachea and esophagus.
REFERENCES: An HS: Principles and Techniques of Spine Surgery. Baltimore, MD,
Williams and Wilkins, 1998, chapter 2.
Flynn TB: Neurologic complication of anterior cervical interbody fusion. Spine 1982;7:536-539.
50. What is the first ossification center to appear radiographically in the pediatric elbow?
1- Trochlea
2- Olecranon
3- Radial head
4- Capitellum
5- Medial epicondyle
PREFERRED RESPONSE: 4
DISCUSSION: The first ossification center to appear in the pediatric elbow is the
capitellum. This ossification center generally appears between the first month and the
11th month in girls and between the first month and the 26th month in boys. The other ossification centers in the elbow appear in the following progression: radial head
(3.8 to 4.5 years), medial epicondyle (5 to 6 years), olecranon (6 to 7 years), trochlea
(9 to 10 years), and the lateral epicondyle (10 years).
REFERENCES: Wilkins KE, Beaty JH, Chambers HG, et al: Fractures and dislocation of the elbow region, in Rockwood CA, Green DP, Bucholz RW, et al (eds): Rockwood and Green’s Fractures in Adults, ed 4. Philadelphia, PA, Lippincott-Raven, 1996, pp 657-662.
Haraldsson S: On osteochondrosis deformans juvenilis capituli humeri including investigation of the intra-osseous vasculature in the distal humerus. Acta Orthop Scand 1959;38:1-232.
51. Following a vertebroplasty of L2, cement is noted to protrude directly anterior to the L2 vertebral body. The cement is closest to which of the following structures?
1- Inferior vena cava
2- Pancreas
3- Duodenum
4- Left lobe of the liver
5- Descending aorta
PREFERRED RESPONSE: 5
DISCUSSION: At the level of L2, the liver and the vena cava lie to the right. The pancreas
and duodenum are anterior to the aorta. The aorta lies in the midline just in front of the
vertebral body.
REFERENCES: Clement CD: Anatomy: A Regional Atlas of Human Anatomy, ed 3.
Baltimore, MD, Munich, Germany, Urban and Schwarzberg, 1987, Figure 331.
Netter FH: Atlas of Human Anatomy. Summit, NJ, Ciba-Geigy, 1989, plate 328.
52. Figures 28a and 28b show AP and lateral radiographs of the knee. Based on these findings, which of the following structures has most likely been injured?
1- Popliteal artery
2- Quadriceps tendon
3- Patellar tendon
4- Anterior cruciate ligament
5- Peroneal nerve
PREFERRED RESPONSE: 4
DISCUSSION: The radiographs show a posterior knee dislocation. Knee dislocations almost always involve rupture of both the anterior and posterior cruciate ligaments. Collateral ligament injuries also are common. Arterial, nerve, and tendon injuries each occur in less than half of knee dislocations.
REFERENCES: Schenck RC Jr, Hunter RE, Ostrum RF, et al: Knee dislocations. Instr Course Lect 1999;48:515-522.
Wascher DC, Dvirnak PC, DeCoster TA: Knee dislocation: Initial assessment and implications for treatment. J Orthop Trauma 1997;11:525-529.
53. A patient who sustained a knife wound to the axilla 4 months ago now has profound interosseous wasting and generalized hand weakness. A brachial plexus injury is likely at which of the following locations in Figure 29?
1- B
2- C
3- K
4- L
5- O
PREFERRED RESPONSE: 4
DISCUSSION: Penetrating sharp wounds in proximity to major nerve or vascular structures should always be acutely explored. Because this patient did not seek treatment for a potentially treatable injury, interosseous wasting implies injury to the C8 and T1 nerve roots that contribute to ulnar nerve function. The most likely location for the brachial plexus injury is the location marked L or the inferior trunk. A wrist drop that is the result of radial nerve dysfunction would be expected with an injury at K or O. An upper brachial plexus palsy with loss of elbow flexion and shoulder abduction would be expected with an injury at B. A loss of elbow flexion alone would be expected following an injury at C.
REFERENCES: Netter F: The Ciba Collection of Medical Illustrations: The Musculoskeletal System. Part 1, Anatomy, Physiology and Metabolic Disorders. West Caldwell, NJ,
Ciba-Geigy, 1991, vol 8, pp 28-29.
Wolock B, Millesi H: Brachial plexus-applied anatomy and operative exposure, in Gelberman RH (ed): Operative Nerve Repair and Reconstruction. Philadelphia, PA, JB Lippincott, 1991, vol 2, pp 1255-1272.
Zimmerman NB, Weiland AJ: Assessment and monitoring of brachial plexus injury in the adult, in Gelberman RH (ed): Operative Nerve Repair and Reconstruction. Philadelphia, PA,
JB Lippincott, 1991, vol 2, pp 1273-1283.
54. During an anterior retroperitoneal approach to the low lumbar spine, the iliac vessels are mobilized along the lateral side, allowing them to be retracted toward the midline. To gain adequate mobility of the common iliac vein for exposure of L5, it is important to identify which of the following structures?
1- Obturator artery
2- Central sacral vessels
3- Internal iliac vein
4- Ascending lumbar vein
5- Iliolumbar vein
PREFERRED RESPONSE: 5
DISCUSSION: The iliolumbar vein is a large tributary that sits along the lateral surface of the common iliac vein. It can be quite substantial in size and must be identified prior to mobilizing the common iliac vein toward the midline. The other structures are not of surgical significance in performing this exposure.
REFERENCE: Gray H: Anatomy of the Human Body. Philadelphia, PA, Lea & Febiger,
1918, 2000.
55. Figure 30 shows an axial T1-weighted MRI scan of a patient’s right shoulder. The arrows are pointing to what normal structure?
1- Deltoid insertion
2- Subscapularis tendon
3- Latissimus dorsi tendon
4- Short head of the biceps tendon
5- Pectoralis major tendon
PREFERRED RESPONSE: 5
DISCUSSION: Tears of the pectoralis major tendon are frequently missed during examination. MRI provides excellent visualization of the tendon if the study extends low enough down the arm. The pectoralis major tendon inserts on the crest of the greater tubercle of the humerus, just lateral to the long head of the biceps tendon. The latissimus dorsi tendon inserts medial to the long head of the biceps tendon on the lesser tubercle. The subscapularis tendon inserts on the lesser tuberosity more proximally. The deltoid insertion is more distal.
REFERENCES: Connell DA, Potter HG, Sherman MF, et al: Injuries of the pectoralis major muscle: Evaluation with MR imaging. Radiology 1999;210:785-791.
Carrino JA, Chandnanni VP, Mitchell DB, et al: Pectoralis major muscle and tendon tears: Diagnosis and grading using magnetic resonance imaging. Skeletal Radiol 2000;29:305-313.
Ohashi K, El-Khoury GY, Albright JP, et al: MRI of complete rupture of the pectoralis major muscle. Skeletal Radiol 1996;25:625-628.
56. The arthroscopic views shown in Figures 31a and 31b reveal extensive synovitis in the anterolateral corner of the ankle overlying a band of tissue sometimes implicated in soft-tissue impingement of the ankle following a chronic sprain injury. This band is a portion of the
1- anteroinferior tibiofibular ligament.
2- anterior talofibular ligament.
3- calcaneofibular ligament.
4- deltoid ligament.
5- extensor retinaculum.
PREFERRED RESPONSE: 1
DISCUSSION: The arthroscopic views show the lateral side of the ankle as demonstrated by the presence of the tibiofibular articulation. As is typical in chronic anterolateral impingement, synovitis overlies the anteroinferior band of the tibiofibular ligament, the most distal portion of the anterior syndesmosis. Hypertrophic scar formed on or in this ligament can impinge on the lateral margin of the talar dome and has been associated with chronic anterolateral ankle pain.
REFERENCES: Bassett FH III, Gates HS III, Billys JB, et al: Talar impingement by the anteroinferior tibiofibular ligament: A cause of chronic pain in the ankle after inversion sprain.
J Bone Joint Surg Am 1990;72:55-59.
DiGiovanni BF, Fraga CJ, Cohen BE, et al: Associated injuries found in chronic lateral ankle instability. Foot Ankle Int 2000;21:809-815.
57. Figures 32a and 32b show the AP and lateral radiographs of an 11-year-old boy who has a severe limp, a fever, and swelling and tenderness of the thigh. Aspiration of the bone reveals purulent material. The patient has most likely been symptomatic for
1- 24 hours.
2- 2 days.
3- 5 days.
4- 7 to 14 days.
5- 6 months.
PREFERRED RESPONSE: 4
DISCUSSION: In patients with an osteomyelitic infection, radiographic findings at 1 to 5 days usually show soft-tissue swelling only. Seven to 14 days after symptoms begin, radiographs will most likely show the classic signs of acute osteomyelitis. Reactive bone formation would be expected by 6 months.
REFERENCES: Kasser JR (ed): Orthopaedic Knowledge Update 5. Rosemont, IL, American Academy of Orthopaedic Surgeons, 1996, pp 149-161.
Song KM, Sloboda JF: Acute hematogenous osteomyelitis in children. J Am Acad Orthop Surg 2001;9:166-175.
58. Figure 33 shows the CT scan of a 40-year-old man who injured his left shoulder while skiing. What structure is attached to the bony fragment?
1- Superior glenohumeral ligament
2- Anterior band of the inferior glenohumeral ligament
3- Middle glenohumeral ligament
4- Subscapularis tendon
5- Long head of the biceps tendon
PREFERRED RESPONSE: 2
DISCUSSION: The scan reveals a bony Bankart lesion. The anterior band of the inferior glenohumeral ligament is the major restraint to anterior translation of the humeral head and is usually injured with anterior shoulder dislocations. It inserts onto the glenoid labrum at the anteroinferior aspect of the glenoid rim. The labrum most frequently avulses from the glenoid (Bankart lesion), but occasionally the bony attachment is avulsed.
REFERENCES: O’Brien SJ, Neves MC, Arnoczky SP, et al: The anatomy and histology of the inferior glenohumeral ligament complex of the shoulder. Am J Sports Med 1990;18:449-456.
Warner JP: The gross anatomy of the joint surfaces, ligaments, labrum and capsule, in Matsen FA, Fu FF, Hawkins RJ (eds): The Shoulder: A Balance of Mobility and Stability. Rosemont, IL, American Academy of Orthopaedic Surgeons, 1992, pp 7-28.
59. What structure is located immediately posterior to the capsule at the posterior cruciate ligament tibial insertion?
1- Popliteal artery
2- Popliteal vein
3- Tibial nerve
4- Peroneal nerve
5- Medial head gastrocnemius tendon
PREFERRED RESPONSE: 1
DISCUSSION: The popliteal artery lies just posterior to the posterior cruciate ligament
tibial insertion, separated only by the posterior capsule of the knee. When performing
a posterior cruciate ligament reconstruction, this artery is at risk for injury during creation
of the tibial tunnel.
REFERENCES: Jackson DW, Proctor CS, Simon TM: Arthroscopic assisted PCL reconstruction: A technical note on potential neurovascular injury related to drill bit configuration. J Arthroscopy 1993;9:224-227.
Malek MM, Fanelli GC: Technique of arthroscopically assisted PCL reconstruction. Orthopedics 1993;16:961-966.
60. A 21-year-old man has mild but persistent aching pain in his left proximal thigh during impact loading activities. He denies pain at rest and has no other symptoms. Figures 34a through 34e show the radiographs and T1-weighted, T2-weighted, and gadolinium MRI scans of the left hip. What is the most likely diagnosis?
1- Enchondroma
2- Giant cell tumor
3- Osteosarcoma
4- Simple bone cyst
5- Histiocytosis
PREFERRED RESPONSE: 4
DISCUSSION: The radiographs show a centrally located radiolucent lesion with cortical thinning and mild osseous expansion; these findings are the hallmarks of a simple bone cyst. Whereas this particular lesion does not demonstrate sclerosis, the distinct margin of this lesion with sharp transition to normal bone is common. The MRI scans reveal a purely cystic lesion with bright T2 signal, and the gadolinium image shows the classic rim enhancement of cystic lesions. Fibrous dysplasia with cystic degeneration might have a very similar appearance and should be considered in the differential diagnosis.
REFERENCES: Parsons TW: Benign bone tumors, in Fitzgerald RH, Kaufer H, Malkani AL (eds): Orthopaedics. St Louis, MO, Mosby, 2002, pp 1027-1035.
May DA, Good RB, Smith DK, et al: MR imaging of musculoskeletal tumors and tumor mimickers with intravenous gadolinium: Experience with 242 patients. Skeletal Radiol 1997;26:2-15.
Resnick D, Kyriakos M, Greenway GD: Tumors and tumor-like lesions of bone: Imaging and pathology of specific lesions, in Resnick D (ed): Diagnosis of Bone and Joint Disorders, ed 4. Philadelphia, PA, WB Saunders, 2002, vol 4, pp 4023-4034.
61. What nerve is at greatest risk when developing the superficial plane between the tensor fascia lata and sartorious during the anterior (Smith-Peterson) approach to the hip?
1- Lateral femoral cutaneous
2- Superior gluteal
3- Inferior gluteal
4- Obturator
5- Femoral
PREFERRED RESPONSE: 1
DISCUSSION: The lateral femoral cutaneous nerve pierces the fascia between the tensor fascia lata and the sartorius approximately 2.5 cm distal to the anterosuperior iliac spine and is at risk when the interval is defined. The superior gluteal and femoral nerves define the internervous plane between the tensor fascia lata and the sartorius and are not at risk for injury.
REFERENCES: Hoppenfeld S, deBoer P: Surgical Exposures in Orthopaedics: The Anatomic Approach. Philadelphia, PA, JB Lippincott, 1984, pp 302-316.
Crenshaw AH (ed): Campbell’s Operative Orthopedics, ed 7. St Louis, MO, CV Mosby, 1987, pp 59-60.
62. An axial T1-weighted MRI scan of the pelvis is shown in Figure 35. Which of the following structures is enclosed by the circle?
1- Obturator vessels and nerve
2- Tendinous origin of the obturator internus muscle
3- Tendinous origin of the obturator externus muscle
4- Seminal vesicle
5- Suprapubic pelvic lymph nodes
PREFERRED RESPONSE: 1
DISCUSSION: The obturator vessels and nerve pass along the lateral pelvic wall along the true pelvic brim (nerve lies anterior to the vessels and lies on the obturator internus muscle) and descend into the obturator groove at the upper portion of the obturator foramen.
REFERENCES: Higuchi T: Normal anatomy and magnetic resonance appearance of the pelvis, in Takahashi HE, Morita T, Hotta T, et al (eds): Operative Treatment of Pelvic Tumors. Tokyo, Japan, Springer-Verlag, 2003, pp 4-21.
Berquist TH: Pelvis, hips and thigh, in Berquist TH (ed): MRI of the Musculoskeletal System,
ed 4. Philadelphia, PA, Lippincott Williams and Wilkins, 2001, pp 210-238.
63. At the level of the midcalf, the plantaris tendon is found at which of the
following locations?
1- Superficial to the fascia of the gastrocnemius
2- Between the medial and lateral heads of the gastrocnemius
3- Between the soleus and lateral head of the gastrocnemius
4- Between the soleus and medial head of the gastrocnemius
5- Between the soleus and posterior tibialis muscle
PREFERRED RESPONSE: 4
DISCUSSION: The plantaris tendon is often harvested to augment a tendon reconstruction. The origin of the plantaris muscle is on the posterolateral aspect of the distal femur, and the muscle lies lateral to the tibial nerve and the posterior tibial artery. The tendon then courses posteriorly between the soleus and the medial head of the gastrocnemius.
REFERENCES: Clement CD: Anatomy: A Regional Atlas of Human Anatomy, ed 3.
Baltimore, MD, Munich, Germany, Urban and Schwarzberg, 1987, Figure 475.
Netter FH: Atlas of Human Anatomy. Summit, NJ, Ciba-Geigy, 1989, plate 491.
64. In the posterior approach to the proximal radius (proximal Thompson approach), the supinator is exposed through the interval between what two muscles?
1- Aconeus and extensor carpi ulnaris
2- Extensor digitorum comminus and extensor carpi ulnaris
3- Extensor digitorum communis and aconeus
4- Extensor carpi radialis brevis and extensor digitorum comminus
5- Extensor carpi radialis brevis and extensor carpi ulnaris
PREFERRED RESPONSE: 4
DISCUSSION: The proximal exposure of the radius is most often used for internal fixation of fractures, resection of tumors, or decompression of the posterior interosseous nerve beneath the supinator muscle. The supinator muscle is exposed through the interval between the extensor carpi radialis brevis and the extensor digitorum comminus muscles. This interval can be more easily palpated further distal in the forearm.
REFERENCES: Hoppenfeld S, deBoer P: Surgical Exposures in Orthopaedics, ed 2. Philadelphia, PA, Lippincott-Raven, 1994, pp 136-146.
Tubiana R, McCullough C, Masquelet A: An Atlas of Surgical Exposures of the Upper Extremity. London, UK, JB Lippincott, 1990, pp 137-141.
65. Figure 36 shows the hip arthrogram of a newborn. Which of the following structures is enclosed by the circle?
1- Limbus
2- Pulvinar
3- Ligamentum teres
4- Transverse acetabular ligament
5- Acetabular labrum
PREFERRED RESPONSE: 5
DISCUSSION: The structure enclosed by the circle is the acetabular labrum. It is visible as the white point of tissue outlined by the darkly radiopaque contrast. The appearance of the contrast surrounding the sharp white point of a normal labrum is called the “rose thorn sign.” The limbus is the term reserved for a rounded, infolded labrum seen with arthrography. The pulvinar is the fatty tissue seen in the empty acetabulum when the hip is dislocated. The ligamentum teres is seen as a white stripe outlined by contrast coursing from the central acetabulum to the dislocated femoral head. The transverse acetabular ligament courses across the inferior portion of the acetabulum and is not clearly seen with arthrography.
REFERENCES: Herring JA: Tachdjian’s Pediatric Orthopaedics, ed 3. Philadelphia, PA,
WB Saunders, 2002, vol 1, pp 532-533.
Severin E: Contribution to the knowledge of congenital dislocation of the hip joint.
Acta Chir Scand 1941;84:1.
66. Figures 37a and 37b show radiographs of a 24-year-old man who has a humeral bone lesion that was found during a screening chest radiograph. He denies any symptoms despite leading a very active lifestyle. What is the most likely diagnosis?
1- Fibrous dysplasia
2- Enchondroma
3- Ewing’s sarcoma
4- Metastatic carcinoma
5- Aneurysmal bone cyst
PREFERRED RESPONSE: 1
DISCUSSION: The radiographs reveal a geographic, diaphyseal lesion with very subtle cortical expansion, cortical thinning, relatively sharp demarcation, and angular rather than rounded borders, suggesting a fibrous bone lesion. This lesion demonstrates the classic ground glass appearance of fibrous dysplasia. Ewing’s sarcoma, metastases, and aneurysmal bone cyst all typically have a more aggressive appearance.
REFERENCES: Parsons TW: Benign bone tumors, in Fitzgerald RH, Kaufer H, Malkani AL (eds): Orthopaedics. St Louis, MO, Mosby, 2002, pp 1027-1035.
Menendez LR (ed): Orthopaedic Knowledge Update: Musculoskeletal Tumors. Rosemont, IL, American Academy of Orthopaedic Surgeons, 2002, pp 69-75.
67. Figures 38a and 38b show the CT scans of a 64-year-old woman. What is the most
likely diagnosis?
1- Calcaneal fibular abutment
2- Symmetrical narrowing of the subtalar joint consistent with an inflammatory arthropathy
3- Cystic lesion of the tibia consistent with enchondroma
4- Stress fracture of the talus
5- A lateral malleolar fracture
PREFERRED RESPONSE: 1
DISCUSSION: The CT scans show large cystic lesions in the talus and calcaneus with complete subluxation of the subtalar joint, allowing the calcaneus to slide laterally until it becomes blocked by the fibula. The cause of this subluxation is severe posterior tibial tendon dysfunction. Although no fibular fracture has yet appeared, it can occur with continued stress from the calcaneus. There is, however, a pathologic fracture in the medial calcaneus through a medial degenerative cyst. The joint space is irregular and not symmetrical as would be seen in an inflammatory arthropathy. Cystic lesions are not present in the tibia. No stress fracture is seen in the talus.
REFERENCES: Coughlin MJ: Sesamoids and accessory bones of the foot, in Coughlin MJ, Mann RA (eds): Surgery of the Foot and Ankle, ed 7. St Louis, MO, Mosby, 1999, pp 437-499.
Anderson RB, Davis WH: Management of the adult flatfoot deformity, in Myerson M (ed):
Foot and Ankle Disorders. Philadelphia, PA, WB Saunders, 2000, pp 1017-1039.
68. The arrow in Figure 39 is pointing to which of the following ligaments?
1- Scapholunate interosseous
2- Lunotriquetral interosseous
3- Ulnolunate
4- Ulnotriquetral
5- Short radiolunate
PREFERRED RESPONSE: 2
DISCUSSION: The lunotriquetral interosseous ligament stabilizes the lunotriquetral joint. The scapholunate interosseous ligament stabilizes the scapholunate joint. The ulnolunate ligament originates from the base of the ulnar styloid and inserts in the lunate. The ulnotriquetral ligament originates from the base of the ulnar styloid and inserts on the triquetrum. The ulnolunate and the ulnotriquetral ligaments are important stabilizers to the ulnar side of the wrist. The short radiolunate ligament originates on the volar ulnar margin of the distal radius and inserts in the ulnar margin of the lunate.
REFERENCES: Berger RA: Ligament anatomy, in Cooney WP, Linscheid RL, Dobyns JH (eds): The Wrist, Diagnosis and Operative Management. St Louis, MO, Mosby, 1998,
pp 73-105.
Adams BD, Divelbiss BJ: Anatomy of the wrist ligaments, in Trumble TE (ed): Carpal Fracture-Dislocatons. Rosemont, IL, American Academy of Orthopaedic Surgeons, 2002, pp 1-5.
69. The medial collateral ligament complex of the elbow originates on what portion of the medial epicondyle?
1- Anterior and inferior
2- Anterior and superior
3- Anterior and central
4- Posterior and superior
5- Posterior and inferior
PREFERRED RESPONSE: 1
DISCUSSION: The medial collateral ligament complex of the elbow consists of three portions: the anterior bundle, the posterior bundle, and a transverse component that has little biomechanic significance. The origin of the ligament is from the central two thirds of the anteroinferior undersurface of the medial epicondyle.
REFERENCE: O’Driscoll SW, Jaloszynski R, Morrey BF: Origin of the medial ulnar collateral ligament. J Hand Surg Am 1992;17:164-168.
70. Figures 40a and 40b show the pre- and postoperative radiographs of an 82-year-old woman with bilateral hip pain who has had staged total hip arthroplasties. To minimize potential injury to the sciatic nerve at the time of surgery, the surgeon should
1- perform a femoral shortening.
2- perform a sciatic neurolysis.
3- release the piriformis tendon.
4- avoid use of posterior acetabular retractors.
5- apply postoperative abduction bracing.
PREFERRED RESPONSE: 1
DISCUSSION: To improve hip biomechanics and secure more suitable bone for acetabular fixation, the true acetabulum is often resurfaced in patients who have developmental dysplasia of the hip, thus lowering the hip center and lengthening the leg. Acute lengthening of more than 3 cm will place excessive tension on the sciatic nerve and require a femoral shortening to avoid sciatic nerve injury. The other maneuvers will not relieve sciatic nerve tension because of
limb lengthening.
REFERENCES: Koval KJ (ed): Orthopaedic Knowledge Update 7. Rosemont, IL, American Academy of Orthopaedic Surgeons, 2002, pp 430-431.
Sanchez-Sotelo J, Berry DJ, Trousdale RT, et al: Surgical treatment of developmental dysplasia of the hip in adults: II. Arthroplasty options. J Am Acad Orthop Surg 2002;10:334-344.
71. Based on the radiographic findings shown in Figure 41, which of the following wrist ligaments is most likely disrupted?
1- Short radiolunate
2- Long radiolunate
3- Scapholunate
4- Ulnolunate
5- Lunotriquetral
PREFERRED RESPONSE: 3
DISCUSSION: The radiograph shows a diastasis of the scapholunate interval, caused by certain failure of the scapholunate interosseous ligament. The lunotriquetral interosseous ligament stabilizes the lunotriquetral joint. The long radiolunate ligament originates in the volar radius and inserts in the lunate. The short radiolunate ligament originates on the ulnar margin of the radius and inserts on the ulnar margin of the lunate. The ulnolunate ligament originates at the ulnar styloid base and inserts on the volar aspect of the lunate.
REFERENCES: Linscheid RL, Dobyns JH, Beabout JW, et al: Traumatic instability of the wrist: Diagnosis, classification, and pathomechanics. J Bone Joint Surg Am 1972;54:1612-1632.
Mayfield JK, Johnson RP, Kilcoyne RK: Carpal dislocations: Pathomechanics and progressive perilunar instability. J Hand Surg Am 1980;5:226-241.
Berger RA: Ligament anatomy, in Cooney WP, Linscheid RL, Dobyns JH (eds): The Wrist, Diagnosis and Operative Management. St Louis, MO, Mosby, 1998, pp 73-105.
72. Which of the following extensor tendons commonly have multiple slips?
1- Extensor pollicis longus and abductor pollicis longus
2- Extensor digiti mini quinti and abductor pollicis longus
3- Extensor digiti mini quinti and extensor carpi radialis brevis
4- Extensor indicis proprius and extensor pollicis longus
5- Extensor indicis proprius and extensor carpi radialis brevis
PREFERRED RESPONSE: 2
DISCUSSION: The extensor digiti mini quinti is most typically a tendon with two slips. The abductor pollicis longus has multiple slips that insert in order of frequency on the base of the first metacarpal, trapezium, and thenar muscles. The extensor pollicis longus, extensor carpi radialis brevis, and extensor indicis proprius consistantly have only one slip.
REFERENCES: von Schroeder HP, Botte MJ: Anatomy of the extensor tendons of the fingers: Variations and multiplicity. J Hand Surg Am 1995;20:27-34.
Bouchlis G, Bhatia A, Asfazadourian H, et al: Distal insertions of abductor pollicis longus muscle and arthritis of the first carpometacarpal joint in 104 dissections. Ann Chir Main Memb Super 1997;16:326-338.
73. The nerve to the abductor digiti quinti, implicated in some patients who have chronic heel pain, is most commonly a branch of what larger nerve?
1- Lateral plantar
2- Lateral calcaneal
3- Medial plantar
4- Medial calcaneal
5- Sural
PREFERRED RESPONSE: 1
DISCUSSION: The nerve to the abductor digiti quinti is the first branch of the lateral plantar nerve. It branches off while the nerve is still on the medial side of the foot and also innervates a portion of the plantar fascia. It can become entrapped beneath the deep fascia of the abductor hallucis muscle and has been associated with some forms of chronic heel pain.
REFERENCES: Baxter DE, Pfeffer GB: Treatment of chronic heel pain by surgical release of the first branch of the lateral plantar nerve. Clin Orthop 1992;279:229-236.
Schon LC, Glennon TP, Baxter DE: Heel pain syndrome: Electrodiagnostic support for nerve entrapment. Foot Ankle 1993;14:129-135.
74. Figure 42 shows the sagittal T2-weighted MRI scan of a patient’s right knee. These findings are most commonly seen with a complete tear of the
1- patellar tendon.
2- lateral collateral ligament.
3- medial collateral ligament.
4- posterior cruciate ligament.
5- anterior cruciate ligament.
PREFERRED RESPONSE: 2
DISCUSSION: The MRI scan reveals disruption of the lateral capsule and ligaments with
fluid in the soft tissues laterally. Additionally, there is a large bone bruise on the medial
femoral condyle. This combination indicates injury to the posterolateral complex. These injuries often have coexisting anterior and/or posterior cruciate ligament injuries. Failure to recognize the posterolateral corner injury can lead to failure of anterior or posterior cruciate ligament reconstructions.
REFERENCES: LaPrade RF, Gilbert TJ, Bollom TS, et al: The magnetic resonance imaging appearance of individual structures of the posterolateral knee: A prospective study of normal knees and knees with surgically verified grade III injuries. Am J Sports Med 2000;28:191-199.
Ross G, Chapman AW, Newberg AR, et al: Magnetic resonance imaging for the evaluation of acute posterolateral complex injuries of the knee. Am J Sports Med 1997;25:444-448.
75. Thermal capsulorrhaphy of the inferior glenohumeral ligament can cause iatrogenic injury to which of the following nerves?
1- Musculocutaneous
2- Suprascapular
3- Radial
4- Axillary
5- Median
PREFERRED RESPONSE: 4
DISCUSSION: The axillary nerve courses from anterior to posterior just below the inferior shoulder capsule. Thermal energy applied to the inferior aspect of the shoulder capsule can result in injury to this nerve.
REFERENCES: Wong KL, Williams GR: Complications of thermal capsulorrhaphy of the shoulder. J Bone Joint Surg Am 2001;83:151-155.
Bryan WJ, Schauder K, Tullos HS: The axillary nerve and its relationship to common sports medicine shoulder procedures. Am J Sports Med 1986;15:113-116.
76. Portions of which of the following normal structures help compose the spiral cord seen in Dupuytren’s contracture?
1- Lateral digital sheet and Grayson’s ligament
2- Lateral digital sheet and Cleland’s ligament
3- Lateral digital sheet and the transverse intermetacarpal ligament
4- Pretendinous band and Cleland’s ligament
5- Intrinsic muscle and the natatory ligament
PREFERRED RESPONSE: 1
DISCUSSION: The normal fascial components that become diseased and compose the spiral cord include the pretendinous band, spiral band, lateral digital sheet, and Grayson’s ligament. Cleland’s ligament lies dorsal and is not involved with spiral cord formation. The intrinsic muscle and tendon, the natatory ligament, and the intermetacarpal ligament are well proximal to the digit and are not involved in spiral cord formation.
REFERENCES: McGrouther D: Dupuytren’s contracture, in Green DP, Hotchkiss RN, Pederson WC (eds): Green’s Operative Hand Surgery, ed 4. New York, NY, Churchill Livingstone,
1999, vol 1, pp 565-569.
McFarlane R: Patterns of diseased fascia in the fingers in Dupuytren’s contracture.
Plast Reconstr Surg 1974;54:31-44.
77. Figure 43 shows an arthroscopic view of the posteromedial compartment of a patient’s left knee using a 70-degree arthroscope placed through the intercondylar notch. The arrow is pointing to what structure?
1- Posterior horn of the medial meniscus
2- Semimembranosus tendon
3- Medial tibial plateau
4- Medial head of the gastrocnemius tendon
5- Medial plica
PREFERRED RESPONSE: 1
DISCUSSION: Passing the 70-degree arthroscope through the intercondylar notch provides excellent visualization of the posteromedial corner of the knee. This view should be part of every knee arthroscopy because these structures are often not well visualized from the anterior portals. If this view is omitted, tears of the peripheral posterior horn of the medial meniscus can be overlooked. The arrow points to the peripheral aspect of the posterior horn of the medial meniscus. With an intact medial meniscus, the medial tibial plateau should not be seen from this view. The semimembranosus and gastrocnemius tendons are extra-articular and not visualized.
REFERENCES: Miller MD: Basic arthroscopic principles, in DeLee JC, Drez D Jr, Miller MD (eds): Orthopaedic Sports Medicine, ed 2. Philadelphia, PA, Saunders, 2003, pp 224-237.
Gold DI, Schaner PJ, Sapega AA: The posteromedial portal in knee arthroscopy: An analysis of diagnostic and surgical utility. Arthoscopy 1995;11:139-145.
78. Which of the following statements best describes the location of the nerve that is at risk in a direct posterior approach to the Achilles tendon?
1- The tibial nerve crosses the Achilles tendon 10 cm above its insertion.
2- The sural nerve crosses the Achilles tendon 5 cm above its insertion.
3- The sural nerve crosses the Achilles tendon 10 cm above its insertion.
4- The superficial peroneal nerve crosses the Achilles tendon 5 cm above its insertion.
5- The superficial peroneal nerve crosses the Achilles tendon 10 cm above its insertion.
PREFERRED RESPONSE: 3
DISCUSSION: The sural nerve lies lateral to the Achilles tendon at the level of the foot but follows an oblique course proximally to lie directly over the tendon as it heads to the popliteal fossa. It is at risk with any proximal dissection from a direct posterior approach and in particular with procedures done at the musculotendinous junction. The nerve crosses over the lateral border of the Achilles tendon at an average of 9.8 cm above its insertion.
REFERENCES: Webb J, Moorjani N, Radford M: Anatomy of the sural nerve and its relation to the Achilles tendon. Foot Ankle Int 2000;21:475-477.
Fletcher MD, Warren PJ: Sural nerve injury associated with neglected tendo Achilles ruptures. Br J Sports Med 2001;35:131-132.
79. A 46-year-old woman has bilateral groin pain, with more severe pain on the left side than on the right side. Figures 44a and 44b show a radiograph and a T1-weighted MRI scan. What is the most likely diagnosis?
1- Rheumatoid arthritis
2- Osteoarthritis
3- Metastatic carcinoma
4- Osteonecrosis
5- Gout
PREFERRED RESPONSE: 4
DISCUSSION: The radiograph reveals bilateral patchy sclerosis of the femoral heads without evidence of collapse. The MRI scan shows bilateral head involvement with a common serpentine-like low-intensity signal of the superior femoral head that is common to osteonecrosis. Changes confined to the femoral head effectively exclude rheumatoid arthritis and osteoarthritis.
REFERENCES: Resnick D (ed): Diagnosis of Bone and Joint Disorders, ed 4. Philadelphia, PA, WB Saunders, 2002, vol 4, pp 3160-3162.
Epstein RE, Dalinka MK: Ischemic necrosis, in Stark DD, Bradley WG (eds): Magnetic Resonance Imaging, ed 3. St Louis, MO, Mosby, 1999, vol 2, pp 1023-1027.
80. Figures 45a through 45c show the radiograph, CT scan, and MRI scan of a 15-year-old boy who has lateral ankle pain. What is the most likely diagnosis?
1- Stress fracture of the calcaneus
2- Rupture of the plantar fascia
3- Early traumatic arthritis of the subtalar joint
4- Calcaneonavicular fibrous coalition
5- Disruption of the os peroneum
PREFERRED RESPONSE: 4
DISCUSSION: The elongated anterior process of the calcaneus reaching distally toward the navicular is an abnormal finding. Instead of viewing the rounded, blunt distal anterior process of the calcaneus, a bridge extends to the navicular, albeit incomplete. These findings are consistent with a fibrous coalition. CT can reveal a stress fracture of the calcaneus, arthritis of the subtalar joint with subchondral cysts, or an os peroneal bone disruption in the peroneus longus, but those entities are not shown here. The plantar fascia is intact.
REFERENCES: Richardson EG: Sesamoids and accessory bones of the foot, in Coughlin MJ, Mann RA (eds): Surgery of the Foot and Ankle, ed 7. St Louis, MO, Mosby, 1999, pp 702-732.
Oestreich AE, Mize WA, Crawford AH, et al: The “anteater nose”: A direct sign of calcaneonavicular coalition on the lateral radiograph. J Pediatr Orthop 1987;7:709-711.
81. A 5-year-old girl has had a low-grade fever, right hip and buttock pain, and a right-sided limp for the past 5 days. Examination shows diffuse tenderness and extreme pain on range of motion of the hip. Laboratory studies show a peripheral WBC count of 13,500/mm3 and an erythrocyte sedimentation rate of 55 mm/h. A radiograph is shown in Figure 46a, and an axial postgadolinium T1-weighted MRI scan with fat suppression and an axial T2-weighted fast spin echo MRI scan are shown in Figures 46b and 46c. What is the most likely diagnosis?
1- Soft-tissue abscess of the gluteii
2- Septic hip
3- Pelvic fracture
4- Acute osteomyelitis of the pelvis
5- Eosinophilic granuloma of the pelvis
PREFERRED RESPONSE: 4
DISCUSSION: MRI findings of acute osteomyelitis include a decrease in the normally high signal intensity of bone marrow on T1-weighted imaging; however, a postgadolinium T1-weighted image with fat suppression will show osteomyelitis as a bright marrow signal compared to the surrounding fat. Osteomyelitis is also brighter than normal fat on T2-weighted imaging. There is no rim-enhancing lesion suggesting an abscess, although myositis is seen in the obturator internus and short external rotators. The clinical scenario and imaging studies do not support the diagnosis of septic hip, eosinophilic granuloma, or pelvic fracture.
REFERENCES: Dormans JP, Drummond DS: Pediatric hematogenous osteomyelitis: New trends in presentation, diagnosis and treatment. J Am Acad Orthop Surg 1994;2:333-341.
Herring JA: Tachdjian’s Pediatric Orthopaedics ed. 3. Philadelphia, PA, WB Saunders, 2002, vol 1, pp 150-163.
82. The palmar cutaneous branch of the median nerve (PCBMN) originates from the
1- radial side of the median nerve and travels radial to the palmaris longus and ulnar to the flexor carpi radialis.
2- radial side of the median nerve and travels distally radial to the palmaris longus and flexor carpi radialis.
3- ulnar side of the median nerve and travels radial to the palmaris longus and ulnar to the flexor carpi radialis.
4- ulnar side of the median nerve and travels radial to the flexor carpi radialis.
5- ulnar side of the median nerve and travels radial to the palmaris longus volar.
PREFERRED RESPONSE: 1
DISCUSSION: The PCBMN originates from the median nerve proper between 3 and 21 cm proximal to the wrist with moderate variation. It virtually always originates from the radial side of the nerve and travels distally with the median nerve, radial to the palmaris longus, and ulnar to the flexor carpi radialis.
REFERENCES: Hobbs RA, Magnussen PA, Tonkin MA: Palmar cutaneous branch of the median nerve. J Hand Surg Am 1990;15:38-43.
Netter F: The Ciba Collection of Medical Illustrations: The Musculoskeletal System: Part 1, Anatomy, Physiology and Metabolic Disorders. West Caldwell, NJ, Ciba-Geigy, 1991, vol 8,
p 52.
83. The blood supply to the anterior cruciate ligament is primarily derived from what artery?
1- Anterior tibial artery
2- Superolateral geniculate
3- Middle geniculate
4- Inferolateral geniculate
5- Inferomedial geniculate
PREFERRED RESPONSE: 3
DISCUSSION: Microvascular studies have shown that the majority of the blood supply to the cruciate ligaments comes from the middle geniculate artery, although there is collateral flow through the other geniculates and from bone.
REFERENCES: Arnoczky SP: Blood supply to the anterior cruciate ligament and supporting structures. Orthop Clin North Am 1985;16:15-28.
Arnoczky SP, Rubin RM, Marshall JL: Microvasculature of the cruciate ligaments and its response to injury. J Bone Joint Surg Am 1979;61:1221-1229.
84. Figures 47a and 47b show the CT scans of a patient who reports persistent pain
in the sinus tarsi following a fall. The avulsion fracture fragment remains attached
to what ligament?
1- Bifurcate
2- Spring
3- Plantar fascia
4- Lisfranc
5- Interosseous
PREFERRED RESPONSE: 1
DISCUSSION: The bifurcate ligament bifurcates to connect the dorsal aspect of the anterior process of the calcaneus to both the cuboid and the navicular. Inversion injuries on the side of the foot can result in avulsion fractures (arrow) of the anterior process of the calcaneus.
REFERENCES: Sarrafian SK: Anatomy of the Foot and Ankle: Descriptive, Topographic, Functional, ed 2. New York, NY, Lippincott, 1993, p 192.
Robbins MI, Wilson MG, Sella EJ: MR imaging of anterosuperior calcaneal process fractures. Am J Roentgenol 1999;172:475-479.
85. Figure 48 shows an MRI scan of the knee. The arrow is pointing to what structure?
1- Iliotibial bone
2- Popliteal tendon
3- Biceps femoris
4- Peroneal nerve
5- Adductor longus
PREFERRED RESPONSE: 3
DISCUSSION: The arrow points to the biceps femoris, which is inserted onto the fibula. The biceps femoris lies at the posterolateral aspect of the thigh. The semimembranosus and the semitendinous lie at the posterior medial aspect of the thigh.
REFERENCES: Gray H: Anatomy of the Human Body. Philadelphia, PA, Lea and Febiger, 1918, 2000.
Netter FH: Atlas of Human Anatomy. Summit, NJ, Ciba-Geigy, 1989, pp 464-465.
86. In Figure 49, line AB connects the anterior arch of C1 to the posterior margin
of the foramen magnum. Line CD connects the anterior margin of the foramen
magnum to the posterior arch of C1. What is the normal ratio of displacement from
CD to AB (Power’s ratio)?
1- 0.25
2- 0.5
3- 1.0
4- 1.5
5- 2.0
PREFERRED RESPONSE: 3
DISCUSSION: The ratio of displacement from CD to AB normally equals 1.0. If the ratio is greater than 1.0, an anterior atlanto-occipital dislocation may exist. Ratios slightly less than 1.0 are normal except in posterior dislocations, fractures of the odontoid process or ring of the atlas, or congenital abnormalities of the foramen magnum. In these conditions, the ratio may
approach 0.7.
REFERENCES: Powers B, Miller MD, Kramer RS, et al: Traumatic anterior atlanto-occipital dislocation. Neurosurgery 1979;4:12-17.
Clark CR: The Cervical Spine, ed 3. Philadelphia, PA, Lippincott-Raven, 1998, pp 50-51.
87. Figure 50 shows the AP radiograph of an asymptomatic 82-year-old woman who underwent total hip arthroplasty 16 years ago. What is the most likely diagnosis?
1- Wear-induced osteolysis
2- Infection
3- Metastatic tumor
4- Loosening of the femoral component
5- Hip subluxation
PREFERRED RESPONSE: 1
DISCUSSION: Pelvic osteolysis in the presence of a well-fixed porous-coated socket is a recognized complication in total hip arthroplasty. The radiograph shows large lytic lesions superiorly adjacent to an acetabular screw and inferiorly extending into the ischium. It also reveals eccentricity of the femoral head with respect to the acetabular component, consistent
with polyethylene wear.
REFERENCES: Koval KJ (ed): Orthopaedic Knowledge Update 7. Rosemont, IL, American Academy of Orthopaedic Surgeons, 2002, p 440.
Dowd JE, Sychterz CJ, Young AM, et al: Characterization of long-term femoral-head-penetration rates: Association with and prediction of osteolysis. J Bone Joint Surg Am 2000;82:1102-1107.
88. The MRI findings shown in Figure 51 would most likely create which of the following signs and symptoms?
1- Weakness of the left extensor hallucis longus and a sensory deficit in the first dorsal web space
2- Diminished right Achilles tendon reflex and weakness of the gastrocnemius-soleus complex
3- Pain radiating into the anteromedial aspect of the left knee, diminished patellar tendon reflex, and difficulty climbing stairs
4- Numbness in the right first dorsal web space and a slap foot gait
5- Urinary retention and right quadriceps weakness with diminished patellar tendon reflex
PREFERRED RESPONSE: 3
DISCUSSION: The MRI scan shows a far lateral disk herniation. With the L4-5 disk, a far lateral herniation abuts the left L4 nerve root. The findings would be consistent with those of a left L4 radiculopathy and would include pain or a sensory deficit on the anteromedial aspect of the knee, diminished patellar tendon reflex, and quadriceps weakness, perhaps making it difficult to walk up and down stairs.
REFERENCES: Fardin DF, Garfin SR (eds): Orthopaedic Knowledge Update: Spine 2. Rosemont, IL, American Academy of Orthopaedic Surgeons, 2002, p 329.
O’Hara LJ, Marshall RW: Far lateral lumbar disc herniation: The key to the intertransverse approach. J Bone Joint Surg Br 1997;79:943-947.
89. Figure 52 shows the MRI scan of a 28-year-old baseball pitcher. Examination will most likely reveal which of the following findings?
1- Weakness of the deltoid
2- Numbness in the C7 dermatomal distribution
3- Winging of the scapula
4- A positive lift-off test
5- Clinical findings similar to a large rotator cuff tear
PREFERRED RESPONSE: 5
DISCUSSION: A ganglion cyst compressing the suprascapular nerve results in poorly localized pain in the shoulder girdle. Sensation is intact, with weakness of external rotation and abduction. Supraspinatus and infraspinatus atrophy is often noted when viewed from behind. These cysts are typically associated with labral tears. Deltoid weakness is associated with an axillary nerve injury, and scapular winging results from injury to the long thoracic nerve.
REFERENCES: Piatt BE, Hawkins RJ, Fritz RC, et al: Clinical evaluation and treatment of spinoglenoid notch ganglion cysts. J Shoulder Elbow Surg 2002;11:600-604.
Inokuchi W, Ogawa K, Horiuchi V: Magnetic resonance imaging of suprascapular nerve palsy.
J Shoulder Elbow Surg 1998;7:223-227.
90. A 23-year-old man has had heel pain and fullness for the past several months. He
reports that initially the pain was present only with activity, but more recently the pain has become constant. Figures 53a through 53d show a radiograph, a bone scan, and
T2-weighted and gadolinium MRI scans. What is the most likely diagnosis?
1- Chondroblastoma
2- Fibrous dysplasia
3- Simple bone cyst
4- Aneurysmal bone cyst
5- Giant cell tumor
PREFERRED RESPONSE: 4
DISCUSSION: The imaging studies reveal an expansile lesion with the classic soap bubble appearance that involves most of the calcaneus. The bone scan reveals a very active lesion with intense uptake, and the MRI scans show the classic, loculated appearance of the lesion with multiple fluid-fluid levels. While it is important to rule out telangiectatic osteosarcoma, the most likely diagnosis is an aneurysmal bone cyst. While giant cell tumor might have a similar appearance, the multiple fluid levels in a expansile lesion strongly favor an aneurysmal
bone cyst.
REFERENCES: Parsons TW: Benign bone tumors, in Fitzgerald RH, Kaufer H, Malkani AL (eds): Orthopaedics. St Louis, MO, Mosby, 2002, pp 1027-1035.
Dorfman HD, Czerniak B: Bone Tumors. St Louis, MO, Mosby, 1998, pp 855-879.
91. A 77-year-old woman who underwent total knee arthroplasty 16 years ago now reports pain, swelling, and notable crepitation with range of motion. AP, lateral, and Merchant radiographs are shown in Figures 54a through 54c. What is the most likely diagnosis?
1- Failure of patellar polyethylene
2- Loosening of the femoral component
3- Loosening of the tibial component
4- Loosening of the patellar component
5- Wear-induced osteolysis
PREFERRED RESPONSE: 1
DISCUSSION: The Merchant radiograph shows a lateral patellar shift with total polyethylene failure, resulting in a metal-on-metal bearing. This problem is associated with metal-backed patellar components. Component fixation appears solid, and no osteolysis is evident.
REFERENCES: Poss R (ed): Orthopaedic Knowledge Update 3. Park Ridge, IL, American Academy of Orthopaedic Surgeons, 1990, pp 590-593.
Leopold SS, Berger RA, Patterson L, et al: Serum titanium level for diagnosis of a failed metal-backed patellar component. J Arthroplasty 2000;15:938-943.
Frymoyer JW (ed): Orthopaedic Knowledge Update 4. Rosemont, IL, American Academy of Orthopaedic Surgeons, 1993, pp 613-614.
92. A 65-year-old woman who works as a florist has had pain in her right elbow for the past 6 months after lifting a flowerpot. MRI scans are shown in Figures 55a and 55b. The area of increased signal intensity seen in Figure 55b most likely represents which of the following findings?
1- Hematoma
2- Edema
3- Soft-tissue sarcoma
4- Antecubital lipoma
5- Brachial artery aneurysm
PREFERRED RESPONSE: 2
DISCUSSION: The MRI scans reveal a chronic distal biceps tendinitis. The T1-weighted scan shows the anatomic detail of the biceps tendon, and the T2-weighted scan shows increased signal caused by edema surrounding the tendon. The T1-weighted scan is not consistent with an antecubital lipoma. The chronicity of the lesion makes hematoma unlikely. An aneurysm usually appears with blood flow through the region and is dark on T1- and T2-weighted scans.
REFERENCES: Morrey BF: Tendon injuries about the elbow, in Morrey BF (ed): The Elbow and Its Disorders, ed 2. Philadelphia, PA, WB Saunders, 1993, pp 492-504.
Morrey B: Distal biceps tendon rupture, in Master Techniques in Orthopedic Surgery:
The Elbow, ed 2. Philadelphia, PA, Lippincott Williams and Wilkins, 2002, pp 173-192.
93. The carpal tunnel canal is narrowest (smallest cross-sectional area) at what level?
1- Proximal to the carpal bones
2- At the level of the lunate
3- At the hook of the hamate
4- At the base of the metacarpals
5- Does not change from proximal to distal
PREFERRED RESPONSE: 3
DISCUSSION: The carpal tunnel canal has an hourglass shape in the coronal plane and is narrowest at the level of the hook of the hamate.
REFERENCES: Cobb TK, Dalley BK, Posteraro RH, et al: Anatomy of the flexor retinaculum. J Hand Surg Am 1993;18:91-99.
Trumble TE (ed): Hand Surgery Update 3: Hand, Elbow and Shoulder. Rosemont, IL, American Society of Surgery of the Hand, 2003, p 300.
94. When harvesting iliac crest bone graft during a posterior spinal decompression and fusion, injury to which of the following nerves may result in painful neuromas or numbness over the skin of the buttocks?
1- Ilioinguinal
2- Superior gluteal
3- Superior cluneal
4- Iliohypogastric
5- Lateral femoral cutaneous
PREFERRED RESPONSE: 3
DISCUSSION: The superior cluneal nerves (L1, L2, and L3) are at greatest risk when harvesting iliac crest bone graft during a posterior decompression and fusion. The nerves pierce the lumbodorsal fascia and cross the posterior iliac crest beginning at 8 cm lateral to the posterior superior iliac spine. The ilioinguinal and iliohypogastric nerves innervate anterior structures, and the lateral femoral cutaneous nerve lies in proximity to the anterior superior iliac spine and is at risk with anterior iliac crest bone graft harvesting. The superior gluteal nerve courses through the sciatic notch and supplies motor branches to the gluteus medius, minimus, and tensor fascia lata muscles.
REFERENCES: An HS: Principles and Techniques of Spine Surgery. Baltimore, MD,
Williams and Wilkins, 1998, pp 770-773.
Kurz LT, Garfin SR, Booth RE: Harvesting autogenous iliac bone grafts: A review of complications and techniques. Spine 1989;14:1324-1331.
95. The injury seen in the CT scan shown in Figure 56 is related to or associated with injury to which of the following structures?
1- Anterior talofibular ligament
2- Calcaneofibular ligament
3- Subtalar joint
4- Syndesmosis
5- Superior peroneal retinaculum
PREFERRED RESPONSE: 4
DISCUSSION: The right syndesmosis appears disrupted on the CT scan when compared to the normal left side. CT can be helpful in determining injury to the syndesmosis, especially with occult clinical findings.
REFERENCES: Ebraheim NA, Lu J, Yang H, et al: The fibular incisure of the tibia on CT scan: A cadaver study. Foot Ankle Int 1998;19:318-321.
Ebraheim NA, Lu J, Yang H, et al: Radiographic and CT evaluation of tibiofibular syndesmotic diastasis: A cadaver study. Foot Ankle Int 1997;18:693-698.
Harper MC: Delayed reduction and stabilization of the tibiofibular syndesmosis. Foot Ankle Int 2001;22:15-18.
96. A 3-year-old girl has had wrist pain, a fever, and has refused to move her right wrist for the past 10 days. She has an oral temperature of 102 degrees F (38.7 degree C). Laboratory studies show an erythrocyte sedimentation rate of 50 mm/h, a WBC count of 11,000/mm3, and a left shift. AP and lateral radiographs are shown in Figures 57a and 57b. What is the most likely diagnosis?
1- Leukemia
2- Eosinophilic granuloma
3- Acute osteomyelitis
4- Ewing’s sarcoma
5- Nonossifying fibroma
PREFERRED RESPONSE: 3
DISCUSSION: The most likely diagnosis is acute osteomyelitis. She may also have a septic wrist; however, the lytic lesion in the distal radius has the typical presentation and radiographic appearance of metaphyseal osteomyelitis. In this area of sluggish vascular flow, low oxygen tension, and low pH, bacterial seeding is common and is the usual origin of metaphyseal osteomyelitis. Leukemia and Ewing’s sarcoma can present as a lytic lesion with an elevated erythrocyte sedimentation rate, but they are much less common than osteomyelitis and are less focal and more destructive in appearance. Nonossifying fibroma is typically metaphyseal and eccentric; however, it is well circumscribed and uncommon in the upper extremity. Eosinophilic granuloma does not typically present with inflammatory indicies.
REFERENCES: Dormans JP, Drummond DS: Pediatric hematogenous osteomyelitis: New trends in presentation, diagnosis and treatment. J Am Acad Orthop Surg 1994;2:333-341.
Song KM, Sloboda JF: Acute hematogenous osteomyelitis in children. J Am Acad Orthop Surg 2001;9:166-175.
97. Which of the following is considered the preferred approach to resect a lesion in the posterior one third of the proximal humerus?
1- Triceps splitting
2- Deltoid splitting
3- Between the lateral head of the triceps and the deltoid muscle
4- Between the long head of triceps and the deltoid muscle
5- Between the long head of triceps and the teres major
PREFERRED RESPONSE: 3
DISCUSSION: At least 8 cm of the posterior aspect of the proximal region of the humeral diaphyseal cortex can be exposed through the interval between the lateral head of the triceps and the deltoid muscle. No nerves or blood vessels need to be exposed in the dissection. The deltoid muscle is innervated by the axillary nerve and the triceps muscle by the radial nerve. This is a true internervous plane.
REFERENCE: Berger RA, Buckwalter JA: A posterior surgical approach to the proximal part of the humerus. J Bone Joint Surg Am 1989;71:407-410.
98. The main blood supply to the lateral two thirds of the talar body is provided by the
1- peroneal artery.
2- anterior tibial artery.
3- anterior lateral malleolar artery.
4- artery of the tarsal sinus.
5- artery of the tarsal canal.
PREFERRED RESPONSE: 5
DISCUSSION: The main blood supply to the lateral two thirds of the talar body is derived from the artery of the tarsal canal, a branch of the posterior tibial artery. The peroneal artery helps form a vascular plexis over the posterior tubercle and combines with other arteries to form the artery of the sinus tarsi, which is the principal blood supply of the intrasinus structures of the talus. The anterior tibial arteries send branches to the superior surface of the talar head and give rise to the anterolateral malleolar artery, which may anastomose with other vessels to form the artery of the tarsal sinus.
REFERENCES: Sanders R: Fractures and fracture-dislocations of the talus, in Coughlin MJ, Mann RA (eds): Surgery of the Foot and Ankle, ed 7. St Louis, MO, Mosby, 1999,
pp 1465-1518.
Haliburton RA, Sullivan CR, Kelly PJ, et al: The extra-osseous and intra-osseous blood supply of the talus. J Bone Joint Surg Am 1958;40:1115-1120.
99. When performing an arthroscopic subacromial decompression, bleeding can be encountered when dividing the coracoacromial ligament because of injury to what artery?
1- Anterior humeral circumflex
2- Acromial branch of the thoracoacromial trunk
3- Clavicular branch of the thoracoacromial trunk
4- Dorsal scapular
5- Subscapular
PREFERRED RESPONSE: 2
DISCUSSION: The acromial branch of the thoracoacromial trunk courses along the coracoacromial ligament. This artery enters the ligament approximately 5 mm below the acromial edge. Division of the ligament at its insertion on the acromion minimizes the risk
of bleeding.
REFERENCES: Esch JC, Baker CL: Arthroscopic anatomy and normal variations, in Whipple TL (ed): Surgical Arthroscopy: The Shoulder and Elbow. Philadelphia, PA, JB Lippincott, 1993, pp 63-76.
Caspari RB, Thal R: A technique for arthroscopic subacromial decompression. J Arthroscopy 1992;8:23-30.
100. Following its exit from the sciatic notch, the sciatic nerve passes between what
two muscles?
1- Piriformis and superior gemellus
2- Superior gemellus and obturator internus
3- Inferior gemellus and quadratus femoris
4- Gluteus medius and minimus
5- Quadratus femoris and gluteus maximus
PREFERRED RESPONSE: 1
DISCUSSION: Though anatomic variations exist, both divisions of the sciatic nerve most commonly pass between the piriformis and superior gemellus. This anatomic consideration is relevant during the posterior approach to the hip, where careful retraction of the rotators avoids sciatic nerve injury.
REFERENCES: Hoppenfeld S, deBoer P: Surgical Exposures in Orthopaedics: The Anatomic Approach. Philadelphia, PA, JB Lippincott, 1984, pp 335-348.
Anderson JE (ed): Grant’s Atlas of Anatomy, ed 7. Baltimore, MD, Williams & Williams,
1978, Figure 4-34.
FOR ALL MCQS CLICK THE LINK ORTHO MCQ BANK